Вы находитесь на странице: 1из 39

ТЕРАПИЯ.

Июнь 2019.

1
I. Тестовое задание.
Определение , этиология , клинические проявления, параклинические исследования ,
осложнения , принципы ухода и лечения .

1. Бронхиальная астма.
Бронхиальная астма— хроническое воспалительное заболевание дыхательных путей.
Ключевым звеном является бронхоспазм (сужение просвета бронхов), обусловленная
специфическими иммунологическими (сенсибилизация и аллергия) или неспецифическими
механизмами, проявляющаяся повторяющимися эпизодами свистящих хрипов, одышки,
чувства заложенности в груди и кашля. Факторы: наследственность, профессиональные,
экологические, микроорганизмы, избыточный вес. Основными симптомами бронхиальной
астмы являются эпизоды одышки, свистящие хрипы, кашель и заложенность в грудной
клетке. Спирометрия, пикфлоуметрия, рентген, бронхоскопия. Серитайт, флекситайт для
лечений, сальбутамол приступ.

2. Пневмония
Воспаление лёгких. «Типичная» пневмония характеризуется резким подъёмом температуры,
кашлем с обильным выделением гнойной мокроты и, в некоторых случаях, плевральной
боли. При исследовании: укорочение перкуторного звука, жёсткое дыхание, усиленная
бронхофония, усиленное голосовое дрожание, сначала сухие, а затем влажные,
крепитирующие хрипы, затемнение на рентгенограмме. Такую пневмонию вызывают
Streptococcus pneumoniae, Haemophilus influenzae, Escherichia coli, Klebsiella pneumoniae.
«Атипичная» пневмония характеризуется постепенным началом, сухим, непродуктивным
кашлем, преобладанием в клинической картине второстепенных симптомов — головной
боли, миалгии, боли и першения в горле, слабости и недомогания при минимальных
изменениях на рентгенограмме. Этот тип пневмонии вызывают, как правило, Mycoplasma
pneumoniae (микоплазменная пневмония), Legionella pneumophila (легионеллёзная
пневмония), Chlamydia pneumoniae (хламидийная пневмония), Pneumocystis jirovecii
(пневмоцистная пневмония). «Вторичная»: аспирационная, септическая, на фоне
иммунодефицита, гипостатическая, посттравматическая и другие. Аспирационная пневмония
— развивается после вдыхания в лёгкие инородной массы (рвотные массы во время
операции, потери сознания, травмы, у новорождённых аспирация амниотической жидкости
во время родов), при этом микробы — возбудители пневмонии — попадают в лёгкие в
составе этой инородной массы. Аспирационная пневмония развивается по типу очаговой
пневмонии. Диагностика Рентгенография грудной клетки Микроскопическое исследования
мокроты с окраской по Граму (Gram) Посев мокроты на питательные среды Общий и
биохимический анализ крови Исследование газового состава крови. Краеугольным камнем
лечения пневмонии являются антибиотики. Выбор антибиотика осуществляется в
зависимости от микроорганизма, вызвавшего пневмонию. Используются также препараты,
расширяющие бронхи и разжижающие мокроту — внутрь или в виде ингаляций,
кортикостероиды, внутривенные солевые растворы, кислород. Иногда выполняются
плевральная пункция и бронхоскопия. Часто используется физиотерапия: ультрафиолетовое
облучение, вибрационный массаж, лечебная физкультура, парафин, озокерит. При
неопределённом типе возбудителя в антибиотикотерапии внебольничной пневмонии
применяют сочетание защищённых пенициллинов и цефалоспоринов (то есть антибиотики
широкого спектра), макролидов; применяются также карбапенемы (тиенам, меропенем),
респираторные фторхинолоны. При неэффективности терапии производят замену

2
антибиотика. Критерием успешности терапии является нормализация температуры тела на
третий день от начала применения антибиотика, а также данные объективного исследования
и рентгенографии грудной клетки.

3. Туберкулез легкого
Туберкулез – это инфекционное заболевание, возбудителем которого являются
микобактерии комплекса Mycobacterium tuberculosis. Пути передачи туберкулезаной
инфекции: аэрогенный, алиментарный, контактный и внутриутробный. Основной путь –
аэрогенный. Источником распространения инфекции являются больные туберкулезом
легких. При кашле, смехе или чихании заразный больной туберкулезом легких выделяет
мельчайшие частицы мокроты, содержащие микобактерии туберкулеза. Долгое время
протекает бессимптомно. Первыми симптомами заболевания становятся признаки общей
интоксикации организма: слабость; вялость; головокружение; плохой аппетит или его
отсутствие; апатия; плохой сон; ночная потливость; бледность; похудение; субфебрильная
(около 37 градусов) температура тела. В случае обращения на этом этапе за медицинской
помощью врач выявит увеличение одной или нескольких групп лимфоузлов –
лимфоаденопатию. При отсутствии лечения начальной стадии заболевания, к
вышеперечисленным присоединяются жалобы со стороны того или иного органа. В случае
поражения легких это: кашель с мокротой или без нее; одышка; боли в грудной клетке при
кашле или в покое; выделение крови с мокротой – кровохарканье. Последние 2 симптома
являются признаками осложненных форм заболевания и требуют немедленного начала
лечения. Рентген лёгких, флюорография, бактериологический анализ мокроты, проба манту.
Одним из самых серьезных осложнений туберкулеза является легочное кровотечение. Кроме
того, возможно развитие спонтанного пневмоторакса и легочной недостаточности.
Начальная стадия лечения, включающует прием 4-х препаратов (рифампицина, изониазида,
пиразинамида и стрептомицина) один раз в день в течение 2 месяцев, а затем –
дополнительную стадию терапии двумя препаратами (рифампицином и изониазидом) в
течение 4 месяцев. При обнаружении высокой степени лекарственной резистентности
производилась замена стрептомицина на этамбутол.

4. Рак легкого
Злокачественное новообразование в тканях лёгких. К основным причинам любого рака
относят канцерогены (например, табачного дыма), ионизирующее излучение и вирусную
инфекцию. Их воздействие вызывает кумулятивные изменения ДНК в выстилающей ткани
бронхов лёгкого (бронхиальном эпителии). Чем больше тканей повреждено, тем выше риск
развития рака. Системная боль в грудной клетке. Центральный рак лёгкого. Опухоль,
происходящая из слизистой оболочки крупного бронха, проявляет себя довольно рано. При
своём росте она раздражает слизистую оболочку бронха, вызывает нарушение бронхиальной
проходимости и вентиляции сегмента, доли или всего лёгкого в виде гиповентиляции и
ателектаза. В дальнейшем прорастая нервные стволы и плевру, опухоль вызывает болевой
синдром и нарушение иннервации соответствующего нерва (диафрагмального, возвратного
или блуждающего), а также картину вовлечения плевры в опухолевый процесс.
Присоединяющееся метастазирование приводит к появлению вторичных симптомов со
стороны поражённых органов и систем. Когда опухоль прорастает в бронх, появляется
кашель, вначале сухой, затем со светлой мокротой, иногда с примесью крови. Возникает
гиповентиляция сегмента лёгкого и затем его ателектаз. Мокрота становится гнойной, что
сопровождается повышением температуры тела, общим недомоганием, одышкой.

3
Присоединяется раковая пневмония, которая сравнительно легко излечивается, но часто
рецидивирует. К раковой пневмонии может присоединиться раковый плеврит,
сопровождающийся болевым синдромом. Если опухоль прорастает возвратный нерв,
присоединяется осиплость ввиду паралича голосовых мышц. Поражение диафрагмального
нерва вызывает паралич диафрагмы. Прорастание перикарда проявляется болями в области
сердца. Поражение опухолью или её метастазами верхней полой вены вызывает нарушение
оттока крови и лимфы из верхней половины туловища, верхних конечностей, головы и шеи.
Лицо больного становится одутловатым, с цианотичным оттенком, набухают вены на шее,
руках, грудной клетке. Периферический рак лёгкого. Периферическая опухоль в начальной
стадии протекает бессимптомно ввиду отсутствия болевых окончаний в лёгочной ткани. В
дальнейшем опухолевый узел увеличивается, прорастает бронхи, плевру и соседние органы;
впоследствии в центре опухоли могут возникнуть распад и кровотечение. При раке лёгкого
могут наблюдаться следующие местные симптомы: кашель, выделение крови с мокротой,
осиплость голоса, синдром сдавливания опухолью верхней полой вены и смещения
средостения, симптомы прорастания опухолью соседних органов. Особенно характерной
клинической картиной, в связи с локализацией, проявляется рак верхушки лёгкого с
синдромом Панкоста. При раковом плеврите присоединяется синдром сдавливания лёгкого
экссудатом. К общим симптомам относятся общее ухудшение состояния организма,
характерное для развития злокачественных опухолей: интоксикация, одышка, слабость,
потеря веса, повышение температуры тела. Для рака лёгкого добавляются также нарушение
обмена кальция, дерматит и деформация пальцев по типу «барабанных палочек». В
запущенных стадиях присоединяются симптомы метастатического поражения жизненно
важных органов, а также присоединяющиеся к опухолевому росту процессы распада
опухоли и лёгочной ткани, бронхиальной непроходимости, ателектазы, тяжёлые лёгочные
кровотечения. Рентген, биопсия опухоли, узи. Лучевая, химиотерапия, хирургическое,
симптоматическое лечение.

5. гипертоническая болезнь
Гипертоническая болезнь – это заболевание, характеризующееся стойким и длительным
повышением артериального давления. ГБ развивается вследствие перенапряжения
психической деятельности под влиянием воздействия психоэмоциональных факторов,
вызывающих нарушение корковой и подкорковой регуляции вазомоторной системы и
гормональных механизмов контроля АД. Эксперты ВОЗ выделяют ряд факторов риска
распространения артериальной гипертонии: возраст, пол, малоподвижный образ жизни,
потребление с пищей поваренной соли, злоупотребление алкоголем, гипокальциевая диета,
курение, сахарный диабет, ожирение, повышенный уровень атерогенных ЛП и
триглицеридов, наследственность и др.
Часто при патологии нет никаких признаков, но иногда могут наблюдаться следующие
симптомы: головная боль; учащенное или слабое биение сердца; головокружения; отеки век,
ног и рук, одутловатость лица; ухудшение памяти; онемение пальцев; покраснение лица;
озноб; повышение потливости; тревога; внутреннее напряжение; снижение
работоспособности; раздражительность; появление «мошек» перед глазами. Три основных
метода диагностики, которые позволяют определить наличие гипертонии у человека, это:
измерение артериального давления, физикальное обследование, электрокардиограмма.
Осложнения - инсульт; - инфаркт; - нарушения зрения; - сердечная недостаточность; -
почечная недостаточность. Лечение При минимальном повышении артериального давления
(ниже 160/100) и при отсутствии дополнительных факторов риска для здоровья достаточно
4
на первых порах устранить избыточный вес тела, ограничить прием поваренной соли и
алкоголя, отказаться от курения, увеличить двигательную активность. Очень важно создать
спокойный психологический климат дома и на работе, быть оптимистичным и
доброжелательным. Эти меры целесообразны также при более значительном повышении
давления и высоком риске сердечно-сосудистых осложнений, однако в этих случаях уже не
обойтись без приема лекарств. Группы препаратов, использующиеся для лечения
артериальной гипертензии: Ингибиторы АПФ («Лизиноприл», «Эналаприл,
«Периндоприл»). Сартаны («Лозартан», «Валсартан»). β-адреноблокаторы («Бисопролол»,
«Метапролол», «Карведилол»). Диуретики («Гипотиазид», «Индапамид», «Верошпирон»).
Антагонисты кальция («Амлодипин», «Нифедипин», «Верапамил»).

6. Острый инфаркт миокарда Острая сердечная патология, характеризующаяся


омертвением определенного участка миокарда, вследствие нарушения его кровообращения.
Данная патология является лидирующей среди причин инвалидизации и смертности среди
населения. Причины острого инфаркта миокарда Ключевая причина развития острой
формы инфаркта миокарда- атеросклеротические поражения венечных артерий, что
приводит к сужению просвета кровеносных сосудов. К провоцирующим факторам следует
отнести: сахарный диабет, гипертония, лишний вес и ожирение, хронические стрессы,
алкоголизм, курение. Симптомы острого инфаркта миокарда Основными симптомами
острой формы данной патологии являются: сильные болевые ощущения, локализированные
в области грудной клетки, которые, как правило, отдают в левое плечо, ухо, челюсть и зубы,
межлопаточную зону. Боли имеют сжимающий, давящий, жгучий, очень острый
«кинжальный» характер. Чем больше зона поражения миокарда, тем более выражена боль.
Боли возникают в виде приступов, для которых характерно усиление или ослабление.
Диагностика острого инфаркта миокарда Основными параметрами для постановки
диагноза служат, классическая клиническая картина заболевания, характерные изменения на
электрокардиограмме больного, а также показатели активности ферментов в сыворотке
крови. В диагностических целях может быть выполнена коронарография. Лечение острого
инфаркта миокарда Больному необходима экстренная медицинская помощь, лечение
проводится в условиях стационара, пациента помещают в кардиологическую реанимацию.
Пациенту необходим полный покой и постельный режим, питание вводится
низкокалорийное малыми порциями. В подостром периоде больного переводят в отделение
кардиологии. В целях купирования болевого синдрома, назначается прием наркотических
анальгетиков (Фентанила) в сочетании с нейролептиками (Дроперидолом), а также вводится
внутривенно Нитроглицерин. В ходе медикаментозной терапии назначаются
антиаритмические средства, ß-адреноблокаторы, препараты тромболитики (Гепарин,
Аспирин), магнезия, нитраты, глюкокортикоиды, спазмолитики. Профилактика острого
инфаркта миокарда Основным методом профилактики является ведение здорового,
регулярная физическая активность, полный отказ от алкоголя и курения, полноценное
питание, исключение стрессов, контроль уровня холестерина крови и показателей
артериального давления.

7. Стенокардия Резкая боль или дискомфорт в области груди, вызванные недостатком


кровоснабжения в определенном участке сердца. Наиболее частая причина развития
стенокардии — атеросклероз коронарных артерий. Значительно реже стенокардия возникает
при инфекционных и инфекционно-аллергических поражениях. Провоцируют приступы
стенокардии эмоциональное и физическое напряжение. Большинство пациентов со

5
стенокардией ощущают дискомфорт или боль в области груди. Дискомфорт обычно
давящего, сжимающего, жгучего характера. Часто боль иррадиирует в левое плечо и
внутреннюю поверхность левой руки, шею; реже — в челюсть, зубы с левой стороны, правое
плечо или руку, межлопаточную область спины, а также в эпигастральную область, что
может сопровождаться диспептическими расстройствами (изжога, тошнота, колики).
Приступ стенокардии обычно продолжается от 1 до 15 минут. Он исчезает при прекращении
нагрузки или приёме нитратов короткого действия (например, нитроглицерина под язык.
ЭКГ, УЗИ сердца, оронарография. Осложнения стенокардии Стенокардия грозна
осложнениями, среди которых: Инфаркт миокарда Аритмии Блокады сердца
Недостаточность кровообращения. нужно принять 1 таблетку нитроглицерина или
использовать спрей с нитроглицерином (нитроминт, изокет). Препараты следует применять
сублингвально (под язык). При отсутствии эффекта необходимо повторить прием препарата
через 2-3 минуты, но применять более 3 доз лекарственного средства нельзя. Если во время
ангинозного приступа произошло повышение кровяного давления, то снижать его
медикаментозными средствами нет необходимости, поскольку после купирования приступа
оно обычно нормализуется самостоятельно. Для лечения и профилактики стенокардии
необходимо устранение факторов риска и модификация образа жизни пациентов: борьба с
ожирением, нормализация массы тела; соблюдение диеты, рекомендованной при
атеросклерозе и заболеваниях сердечно-сосудистой системы; отказ от вредных привычек
(курение, употребление алкоголя); регулярные физические нагрузки (необходимо
согласование с врачом); нормализация психоэмоционального состояния; соблюдение
рекомендаций врача по лечению сердечно-сосудистых и других заболеваний; регулярные
профилактические осмотры у терапевта и кардиолога (особенно для лиц старше 45 лет).
Медикаментозная профилактика приступов стенокардии заключается в систематическом
применении нитратов пролонгированного действия или других антиангинальных средств.
При стабильном течении стенокардии прием нитропрепаратов необходим только перед
физическими нагрузками.

8. острая сосудистая недостаточность (липотимия, синкопа, коллапс, шок)


Липотимия - обморочная реакция. Липотимия характеризуется внезапным легким
затуманиванием сознания, головокружением, звоном в ушах, тошнотой, похолоданием рук и
ног. Объективно отмечается резкая бледность кожных покровов, легкий цианоз губ,
расширение зрачков, малый пульс, снижение артериального давления. Приступ липотимии
длится несколько секунд. Собственно обморок начинается с симптомов липотимии, за
которым следует потеря сознания. Больной медленно падает (оседает). Пульс малый или
совсем не определяется. Артериальное давление резко снижено, дыхание поверхностное,
сухожильные и кожные рефлексы не вызываются. Длительность потери сознания 10 – 30
секунд. После обморока некоторое время сохраняется общая слабость, тошнота, дискомфорт
в брюшной полости.
Коллапс — угрожающее жизни состояние, характеризующееся падением кровяного давления
и ухудшением кровоснабжения жизненно важных органов. У человека проявляется резкой
слабостью, заостренными чертами лица, бледностью, похолоданием конечностей. Возникает
при инфекционных болезнях, отравлениях, большой кровопотере, передозировке, побочных
действиях некоторых препаратов и др.
Шок — патологический процесс, развивающийся в ответ на воздействие чрезвычайных
раздражителей и сопровождающийся прогрессивным нарушением жизненно важных

6
функций нервной системы, кровообращения, дыхания, обмена веществ и некоторых других
функций. По сути, это срыв компенсаторных реакций организма в ответ на повреждение.

9. Хроническая сердечная недостаточность


Хроническая сердечная недостаточность — заболевание, сопровождающееся характерными
симптомами (одышка, снижение физической активности, утомляемость, отеки и др.),
связанными с неадекватной перфузией органов и тканей в покое или при нагрузке,
сопровождающееся с задержкой жидкости в организме. К основным причинам развития
ХСН относятся: артериальная гипертензия, ишемическая болезнь сердца, перенесенный
инфаркт миокарда, сахарный диабет. Комбинация ишемической болезни сердца и
артериальной гипертензии встречается у большинства больных ХСН. Типичные симптомы и
признаки ХСН: одышка, слабость, повышенная утомляемость, сердцебиение, ортопноэ,
отеки. Специфические признаки ХСН: набухание шейных вен, гепатоюгулярный рефлюкс,
третий тон сердца (ритм галопа), смещение верхушечного толчка влево. Анализ обращений
больных с симптомами ХСН показал, что в 63 % случаев пациенты обращаются за помощью
лишь тогда, когда ухудшение становится клинически значимым и требует госпитализации и
стационарного лечения. Проявления начальных стадий ХСН, особенно у больных с АГ и
ИБС, недооцениваются. ЭКГ, клинические симптомы, УЗИ. Лечится альфа/бетта
блокаторами, антиагригантами.

10. Хронический гастрит


Гастрит — воспалительные или воспалительно-дистрофические изменения слизистой
оболочки желудка. Способствующими возникновению хронического гастрита, являются:
заражённость желудка Helicobacter pylori и, в меньшей степени, другими бактериями или
грибами; нарушения питания; вредные привычки: алкоголизм и курение; длительный приём
лекарств, раздражающих слизистую оболочку желудка, в особенности, глюкокортикоидных
гормонов и нестероидных противовоспалительных препаратов, ацетилсалициловой кислоты;
воздействие на слизистую радиации и химических веществ; паразитарные инвазии
(описторхоз, лямблиоз и т.п.); хронический стресс. Внутренними (эндогенными) факторами,
способствующими возникновению хронического гастрита, являются: генетическая
предрасположенность; дуоденогастральный рефлюкс; аутоиммунные процессы,
повреждающие клетки желудка эндогенные интоксикации; гипоксемия; хронические
инфекционные заболевания; нарушения обмена веществ; эндокринные дисфункции;
недостаток витаминов; рефлекторные воздействия на желудок от других поражённых
органов. Местные расстройства характеризуются симптомами диспепсии (тяжесть и чувство
давления, полноты в подложечной области, появляющиеся или усиливающиеся во время еды
или вскоре после еды, отрыжка, срыгивание, тошнота, неприятный привкус во рту, жжение в
эпигастрии, нередко изжога, которая говорит о нарушении эвакуации из желудка и забросе
желудочного содержимого в пищевод). Эти проявления чаще возникают при определенных
формах хронического антрального гастрита, которые ведут к нарушению эвакуации из
желудка, повышению внутрижелудочного давления, усилению гастроэзофагеального
рефлюкса и обострению всех перечисленных симптомов. При хроническом гастрите тела
желудка проявления встречаются нечасто и сводятся преимущественно к тяжести в
эпигастральной области, возникающей во время или вскоре после еды. Общие расстройства
могут проявляться следующими синдромами: слабостью, раздражительностью,
нарушениями со стороны сердечно-сосудистой системы — кардиалгии, аритмии,
артериальной неустойчивостью; у пациентов с атрофическим хроническим гастритом может

7
развиться симптомокомплекс, схожий с демпинг-синдромом (внезапная слабость, бледность,
потливость, сонливость, возникающие вскоре после еды), иногда сочетающихся с
расстройствами кишечника, с императивным позывом к стулу; у пациентов с хроническим
гастритом тела желудка и развитием В12-дефицитной анемии появляются слабость,
повышенная утомляемость, сонливость, наблюдается снижение жизненного тонуса и утрата
интереса к жизни; возникают боли и жжение во рту, языке, симметричные парестезии в
нижних и верхних конечностях; у пациентов с Helicobacter pylori-ассоциированным
антральным хроническим гастритом с повышенной кислотностью возможно развитие
язвенноподобных симптомов, свидетельствующих о возможном предъязвенном состоянии.
Клиническая диагностика, ФГДС, УЗИ. Лечение включает как медикаментозные, так и
немедикаментозные методы. К основным видам препаратов относятся: антациды (альмагель,
фосфалюгель), обволакивающие средства(висмута трикалия дицитрат), антибактериальные
средства для эрадикации хеликобактер пилори, блокаторы гистаминных рецепторов
(ранитидин), ингибиторы протонного насоса (омепразол), ферментные препараты (фестал,
креон), противорвотные препараты и прокинетики.

11. Язвенная болезнь желудка и двенадцатиперстной кишки


Этиология: плохая наследственность; неправильное питание: употребление грубой и острой
пищи. При этом повышается выработка соляной кислоты. злоупотребление алкоголем.
Алкоголь усиливает агрессивные свойства желудочного сока и снижает защитные свойства
слизистой оболочки. курение. Никотин усиливает выработку соляной кислоты, препятствует
нормальному перевариванию пищи, повреждает стенку желудка, а также нарушает
выработку факторов защиты слизистой желудка в поджелудочной железе.; бесконтрольный
прием некоторых лекарств (резерпин, кортикостероидные гормоны, аспирин). Беспокоят
ночные и «голодные» боли, при которых человеку необходимо что-нибудь съесть, чтобы
«погасить» боль. Боли при язвенной болезни имеют четкую ритмичность (время
возникновения и связь с приемом пищи), периодичность (чередование болевых ощущений с
периодами их отсутствия) и сезонность обострений (весной и осенью). Характерно, что боли
при язвенной болезни уменьшаются или исчезают после приема пищи и антацидов
(альмагель, маалокс). Одним из частых симптомов язвенной болезни является изжога,
которая обычно возникает спустя 2-3 часа после еды. Тошнота, рвота, «кислая» отрыжка,
запоры – эти неспецифические симптомы также могут свидетельствовать о язве. Аппетит
при язвенной болезни обычно сохранен или даже повышен, так называемое «болезненное
чувство голода». Эндоскопия: язва желудка — это резко ограниченный, круглый дефект
диаметром ≈1 см или неправильной формы углубление с инфильтрированными краями, чаще
всего в области угла желудка или препилорической области, обычно одиночное;
множественные язвы нередко диагностируемые после приема НПВП. В
двенадцатиперстной кишке язва чаще всего на передней стенке луковицы, как правило,
диаметром <1 см. Срочным показанием к эндоскопии является кровотечение из верхнего
отдела пищеварительного тракта. Тесты, выявляющие инфекцию H. pylori (перед
выполнением теста кроме серологического отменить антибиотики и висмут на ≥4 недели, а
ИПП на 2 нед). Показания к тестированию. 1) инвазивные методы (нуждающиеся в
выполнении эндоскопии): а) уреазный тест (используется чаще всего), биоптат слизистой
оболочки желудка помещается на пластину, содержащую мочевину с добавлением цветового
индикатора, разложение бактериальной уреазой мочевины до аммиака подщелачивает среду
и приводит к изменению ее цвета (чувствительность и специфичность 95 % при анализе 2
биоптатов); б) гистологическое исследование биоптата слизистой оболочки; в)
8
культивирование бактерий. 1. Диета: регулярный прием пищи, с исключением только блюд,
вызывающих или усиливающих жалобы. Ограничить употребление кофе и крепкого
алкоголя (хотя нет доказательств, чтобы это помогало в заживлении язв). Алкоголь и метод
питания не влияют на возникновение пептических изъязвлений. 2. Отказ от табакокурения:
курение затрудняет заживление язвы и увеличивает риск рецидива. ингибиторы протонной
помпы (омепразол, квамател) антибактериальные средства, ДИЕТА.

12. Хронический панкреатит


Панкреатит – заболевание поджелудочной железы воспалительного характера. Причины:
неинфекционный гепатит; желчнокаменная болезнь; гастрит; цирроз печени; дискинезия
желчевыводящих путей; кишечные инфекции. У больного наблюдаются постоянные,
рецидивирующие боли в верхних отделах живота, длительные по характеру и
усиливающиеся после приема жирной, острой и жареной пищи. Возникают диспептические
явления - вздутие, урчание в животе, тошнота, отрыжка, рвота. Наблюдается постепенное
снижение массы тела, диарея, стул у больного обильный и зловонный, консистенция
каловых масс кашицеобразная. У больного отмечается пожелтение кожи, слизистых
оболочек и склер глаз, вследствие повышения уровня билирубина. Осложнение: сахарный
диабет, развивающийся как следствие недостаточности поджелудочной железы, снижении
выработки гормона инсулина; общее истощение, хроническая интоксикация организма;
абсцесс поджелудочной железы; легочные осложнения; развитие панкреатогенного асцита;
болезни и дисфункции иных органов желудочно-кишечного тракта (холецистит, язвенная
болезнь двенадцатиперстной кишки); кистозные образования в тканях железистого органа;
механическая желтуха как следствие сдавливания желчных протоков; летальный исход.
Диагностика: общий анализ кала, узи, рентген. Применяют витаминотерапию: медикаменты
и лекарственные средства с витаминами А, С, Д, К, Е и группы В, назначают прием липоевой
кислоты, кокарбоксилазы и так далее по показаниям. Важной составляющей лечения
является диета со строгим исключением жирной, острой пищи, категорически запрещается
употребление алкоголя.

13. Хронический гепатит Дистрофическое поражение печени, воспалительной природы,


характеризующееся длительным течением. У больного отмечается увеличение размеров
печени, при пальпации она четко прощупывается. В области печени возникает тупая боль,
возникает брадикардия, общее психологическое состояние ухудшается, человек становится
раздражительным. Возникает бессонница, тошнота, расстройства желудка, отрыжка,
метеоризм, повышенная утомляемость, аппетит снижается, отмечается появление
расширенных капилляров на лице, появление характерно признака «печеночные ладони»,
возникают носовые кровотечения, боли в мышцах и суставах. Проводится анализ анамнеза
заболевания и жизни больного, в целях постановки диагноза выполняется: биохимический
анализ крови, биопсия печени, анализ крови на определение антител к вирусам гепатита,
вирусологические и иммунологическое исследования. Пациенту следует соблюдать строгую
диету (стол №5), исключить алкоголь, кофейные напитки и чай, максимально сократить
физические нагрузки, достаточное количество времени отдыхать. В ходе лекарственной
терапии больному назначаются препараты иммуномудоляторы, противовирусные средства
(интерферон-ɑ, ламивудин), гепатопротекторы (эссенциале, гепабене, гептал). Существует
высокая вероятность развития осложнений, в особенности, при проведении некорректного
лечения, таких как: цирроз печени, гепатоцеллюлярная карцинома.

9
14. Цирроз печени
Цирроз печени — хроническое заболевание печени, сопровождающееся необратимым
замещением паренхиматозной ткани печени фиброзной соединительной тканью, или
стромой. Длительная интоксикация алкоголем, гепатиты. Для цирроза характерны общие
симптомы: слабость, пониженная трудоспособность, неприятные ощущения в животе,
диспепсические расстройства, повышение температуры тела, боли в суставах, также
отмечаются метеоризм, боль и чувство тяжести в верхней половине живота, похудение,
астения. При осмотре выявляется увеличение печени, уплотнение и деформация её
поверхности, заострение края. Сначала отмечается равномерное умеренное увеличение
обеих долей печени, позднее как правило преобладает увеличение левой доли. Портальная
гипертензия проявляется умеренным увеличением селезенки. Развёрнутая клиническая
картина проявляется синдромами печёночно-клеточной недостаточности и портальной
гипертензии. Имеют место вздутие живота, плохая переносимость жирной пищи и алкоголя,
тошнота, рвота, диарея, чувство тяжести или боль в животе (преимущественно в правом
подреберье). В 70 % случаев обнаруживается гепатомегалия, печень уплотнена, край
заострён. У 30 % больных при пальпации выявляется узловатая поверхность печени.
Спленомегалия у 50 % больных. Субфебрильная температура, возможно, связана с
прохождением через печень кишечных бактериальных пирогенов, которые она не в
состоянии обезвредить. Лихорадка резистентна к антибиотикам и проходит только при
улучшении функции печени. Могут быть внешние признаки — пальмарная или плантарная
эритема, сосудистые звёздочки, скудный волосяной покров в подмышечной области и на
лобке, белые ногти, гинекомастия у мужчин вследствие гиперэстрогенемии. В ряде случаев
пальцы приобретают вид «барабанных палочек». Осложнения печёночная кома кровотечение
из варикозно-расширенных вен пищевода тромбоз в системе воротной вены
гепаторенальный синдром формирование рака печени — гепатоцеллюлярной карциномы
инфекционные осложнения — пневмонии, «спонтанный» перитонит при асците, сепсис.
Цирроз печени с диспропорциональной гипертрофией сегментов. Компьютерная томография
брюшной полости. Цирроз печени с уменьшением объёма органа. Компьютерная томография
брюшной полости. Характерно повышение активности АЛТ, АСТ, щелочной фосфатазы,
лейкоцитоз. При гепатолиенальном синдроме может развиваться гиперспленизм,
проявляющийся лейкопенией, тромбоцитопенией, анемией и увеличением клеточных
элементов в костном мозге. Расширенные и извитые венозные коллатерали выявляются при
ангиографии, компьютерной томографии, ультразвуковом исследовании или в ходе
оперативного вмешательства. Строгая диета, обезболивающие, отказ от вредных привычек.

15. Хронический холецистит


Хронический холецистит — воспаление желчного пузыря, развивающееся медленно и
постепенно. Желчнокаменная болезнь, паразиты, бактерии. При хроническом холецистите
человек периодически ощущает: тупые боли в правом подреберье вздутие живота; тошноту,
рвоту; нарушение переваривания жирной пищи (поносы после еды). ОАК, БИОХИМИ
КРОВИ, УЗИ, холоцистохолангиография. Диета, желчегонные препараты (холосас,
холензим, холагол, лиобил и др.); отвары желчегонных трав (кукурузные рыльца, цветки
бессмертника и др); спазмолитики (в частности, но-шпа, баралгин) – при болях; в ряде
случаев – антибиотики и противовоспалительные средства. Дуоденальные зондирования.

10
16. Острый и хронический пиелонефрит
Пиелонефрит – заболевание почек инфекционной этиологии, сопровождающееся
воспалительным процессом. Причины пиелонефрита в проникновении и размножении в
тканях почек патогенных бактерий, находящихся в нижних отделах мочевыделительной
системы (мочевом пузыре). Выделяют также гематогенный, лимфогенный пути заражения и
внесение инфекционного агента при травмах или медицинских манипуляциях. Клиника
резкое повышение температуры тела до фебрильных показателей (свыше 38°С); симптомы
интоксикации организма: тошнота, озноб, лихорадка, головные боли; возможное изменение
характеристик мочи, особенно при одновременном наличии цистита: потеря прозрачности,
наличие включений крови, гноя и т. п. умеренными болевыми ощущениями в области
поясницы; незначительными явлениями интоксикации: слабостью, головными болями,
снижением аппетита; отечность после ночного сна при наличии затруднения с выделением
мочи. неприятные ощущения, боль, жжение, сопровождающие мочеиспускание; частые
позывы в туалет, ощущение неполного опорожнения мочевого пузыря; боли внизу живота и
т. д. Диагностика сбор анамнеза со слов пациента и информации из медицинских
документов: наличие текущих или имевшихся острых и хронических инфекционных
заболеваний, патологий строения, болезней мочеполовой системы; сбор информации для
составления клинической картины заболевания: жалобы на боли в области поясницы, вид,
запах мочи, частоту мочеиспускания, гипертермию, признаки общего недомогания; осмотр
пациента;лабораторные анализы, инструментальная диагностика. В перечень лабораторных
анализов относят следующие исследования: общий и биохимический анализ крови, мочи;
пробы по Зимницкому и Нечипоренко; посев микрофлоры мочи для определения
чувствительности к антибиотикам разных групп. Длительная терапия антибиотиками,
нацеленными на излечение воспалительных процессов в почках, без внимания к очагам
инфекции приводит к потере чувствительности патогенных бактерий и отсутствию эффекта
от приема препаратов. Чаще всего пиелонефрит возникает при размножении кишечной
палочки, возбудителя, хорошо поддающегося воздействию полусинтетическим
пенициллином в комбинации с клавуновой кислотой (Аугментин), цефалоспоринами 1-3
поколения. В качестве противомикробных средств наиболее распространены
фторхиналоновая группа (Нолицин), ципрофлоксацины (Ципрофлоксацин, Цифран). Для
профилактики рецидивов при хронической форме больным пиелонефритом используют
производные налидиксовой кислоты (Невиграмон) и оксихинолина (5-НОК).

17. Острый и хронический гломерулонефрит


ОСТРЫЙ ГЛОМЕРУЛОНЕФРИТ (ОГН) - диффузное воспалительное заболевание почек с
преимущественным поражением клубочков. Основным этиологическим фактором ОГН
являются стрептококк, главным образом гемолитический типа группы А. Клиническая
картина. отечный, гипертензивный и мочевой синдромы. ОГН начинается остро, бурно,
четко связан с перенесенной ангиной, обострением хронического тонзиллита, фарингита или
другой очаговой стрептококковой инфекцией, предшествовавшей возникновению мерных
признаков заболевания за 1-3 нед. На первый план выступают головная боль, слабость,
общее недомогание, снижение аппетита, одышка, сердцебиение, боли в области сердца,
поясницы, в ряде случаев учащенное и болезненное мочеиспускание, малое количество и
изменение цвета мочи, появление отеков. Отмечается бледность кожных покровов,
одутловатость лица. В тяжелых случаях больные принимают вынужденное сидячее или
полусидячее положение в постели вследствие явлений острой (чаще левожелудочковой)
сердечной недостаточности. Наблюдаются цианоз губ, акроцианоз, дыхание учащенное и
11
глубокое, затрудненное. Диагноз УЗИ, биохимия крови, выявления вирусов в крови, моче.
Осложнения. Течение ОГН в тяжелых случаях может осложняться ОПН, острой сердечной
недостаточностью и эклампсией. Симптоматическая терапия, диуретики, антигистаминные
средства, антибиотики.

18. хроническая почечная недостаточность .


ХПН - патологическое состояние организма, характеризующееся постоянным
прогрессирующим нарушением функции почек. Этиология: Хронический гломерулонефрит;
- Хронический пиелонефрит (есть мнение, что основная причина) - Поликистоз почек; -
Злокачественная гипертоническая болезнь с исходом в нефросклероз; - Амилоидоз почек; -
Разнообразная урологическая патология (мочекаменная болезнь и др.); - Диабетический
гломерулосклероз; - Аденома и рак предстательной железы (последствие уростаза). Со
стороны нервной системы: слабость, повышенная утомляемость, потеря интереса к
окружающему, потеря памяти, днем сонливость, вечером бессонница, понижение аппетита,
иногда упорная рвота, головные боли, иногда кожный зуд. Со стороны сердечно -
сосудистой системы: гипертония до 200 - 140/130 - 140 мм рт. ст. ОАК(анемия), УЗИ, анализ
мочи. Диуретики, натрия/кальция бикарбонат, гемодиализ.

19. Железодифицитная анемия


Железодефици́тная анемия (ЖДА) — гематологический синдром, характеризующийся
нарушением синтеза гемоглобина вследствие дефицита железа.Этиология: нарушение
баланса в сторону преобладания расходования железа над поступлением, наблюдаемое при
различных физиологических состояниях или заболеваниях:* кровопотери различного
генеза;* повышенная потребность в железе;* нарушение усвоения железа;* врождённый
дефицит железа;нарушение транспорта железа вследствие дефицита трансферрина.
Клиника:Анемический синдром обусловлен тканевой гипоксией, его проявления
универсальны для всех видов анемий* слабость и/или быстрая утомляемость;* бледность
кожи и слизистых; головная боль и/или пульсация в висках;* головокружение, обмороки; *
одышка и сердцебиение при привычной физической нагрузке;* усиление ангинозных болей
при ИБС; Сидеропенический синдром обусловлен тканевым дефицитом Fe. Основные
проявления:* сухость кожи, трещины на поверхности кожи рук и ног, в углах рта глоссит,
сопровождающийся атрофией сосочков, болезненностью и покраснением языка;* ломкость,
истончение, расслоение ногтей, ногти ложкообразной формы; * выпадение волос и раннее их
поседение;* извращение вкуса пациенты едят мел, уголь, глину, песок и/или сырые
продукты (крупы, фарш, тесто); * пристрастие к необычным запахам (керосина, мазута,
бензина, ацетона) Диагностика анализ крови. Из показателей: гемоглобин
(снижено),эритроциты (снижено),Цветной показатель (ниже нормы – гипохромная
анемия),При изучении морфологии (анизоцитоз, пойкилоцитоз). В биохимическом анализе
указывают:уменьшение количества сывороточного железа, уменьшение насыщения железом
трансферрина.,Уменьшение количества в сыворотке ферритина) Повышение общей
железосвязывающей функции сыворотки крови. Далее врачи гематологи назначают целый
комплекс обследований и консультаций с целью установления причины анемии.
Лечение::устранение причины, ее вызвавшей , восполнение дефицита Fe в организме;
предотвращение развития дистрофических изменений внутренних органов и сохранение их
функциональной способности в полном объеме.Диета-один из вспомогательных
компонентов терапии (продукты, содержащие достаточное количество хорошо всасываемого
белка и Fe (печень, яйца, рыба), (бобовые, соя, шпинат, укроп, салат, абрикосы, чернослив,

12
хлеб, рис)),Гемотрансфузии проводятся пациентам только по жизненным показаниям (общее
состояние пациента и гемодинамики). Медикаментозная терапия ЖДА: * назначение
препаратов Fe с достаточным содержанием в них двухвалентного Fe2+ (200–300 мг/сутки)
совместно с веществами, усиливающими их всасывание (аскорбиновая и янтарная кислота);

20. B12 - дифицитная анемия


Мегалобластная анемия– заболевание, характеризующееся нарушением кроветворения,
возникающим из-за дефицита витамина В12 в организме. Этиология : вызывается
недостатком в организме человека витамина В12 или фолиевой кислоты, которые наряду с
железом принимают активное участие в синтезе эритроцитов. Развиваться дефицит фолиевой
кислоты и витамина В12, чаще при недостатке их в употребляемой пище или во время
некоторых заболеваний желудочно-кишечного тракта, врожденное отсутствие витамина В12-
связывающих компонентов. Клиника : гипоксия тканей, жалобы на общую слабость,
изменения обоняния и вкуса вялость, головную боль, головокружение, сонливость,
систолический шум, одышку, бледность, тахикардия, боли в области сердца, гипотония,
нарушение желудочно-кишечного тракта в виде алхилии, глоссита, невриты, радикулитыы,
полиневриты. Диагностика : осмотр пациента, жалобы пациента ,анамнез заболевания,
ОАК(уровень эритроцитов и гемоглобина снижен), миелограмма, При пальпации живота -
увеличение в размерах селезенки( спленомегалия), В сыворотке крови снижен показатель
витаминов B9 и B12., в сыворотке повышен по сравнению с нормой уровень свободного
билирубина, биохимический анализ крови, пункция костного мозга Лечение : коррекция
питания, диета витаминизированная включающая продукты богатые
железом(говядина,телятина,печень),дегельминтизация, лечение других заболеваний,
явившихся причиной мегалобластной анемии., назначается витамин В12 в\м по 200-500мкг в
сутки, препататы содержащие железо(гемофер, ферроплекс, конферон, дурулес которые
принимаются после еды 2-3 раза в день в течении 2-3 месяцев.

21. Дифузный токсичесикй зоб


ДТЗ - Гипертрофия и гиперфункция щитовидки, при которых развивается тиреотоксикоз.
Причины- дефекты иммунной системы, влияющие на выработку антител к рецепторам
тиреотропного гормона. генетическая предрасположенность. на фоне инфекционных и
воспалительных процессов, психических травм, органических поражений мозга, черепно-
мозговых травм, энцефалита, аутоиммунных и эндокринных расстройств, пагубных
привычек (курение). Симптомы У больных наблюдаются сердечно-сосудистые изменения,
катаболический синдром и эндокринная офтальмопатия. Основными
признаками :тахикардия, повышенное систолическое и пониженное диастолическое
давление. Иногда у больных к вечеру поднимается субфебрильная температура.
Эндокринная офтальмопатия выражается выпученными глазами, редким миганием, блеском
глаз. Диагностика - врач анализирует симптомы, изучает историю болезни, проводит
физикальный осмотр и назначает дополнительные обследования. Как правило, выполняют
анализ крови на уровень гормонов щитовидки и гипофиза (Т3, Т4, ТТГ), иммуноферментный
анализ крови, ультразвуковое исследование, сцинтиграфию, рефлексометрию. Лечение-
Консервативная терапия включает антитиреоидные препараты (Мерказолил, Метизол,
Тирозол, Пропицил). По показаниям выполняют оперативное вмешательство, во время
которого щитовидную железу почти полностью удаляют. В некоторых случаях используется
радиойодтерапия. Тиреотоксический криз лечат тиреостатиками, глюкокортикоидами и b-
аденоблокаторами. При тяжелом течении недуга возможно выполнение плазмафереза.

13
Профилактика- Специфические меры профилактики не разработаны. Если в семейном
анамнезе имеется диффузно токсический зоб, необходимо регулярно проходить
эндокринологические исследования.

22. гипотиреоз
Гипотиреоз - гетерогенный синдром, характеризующийся снижением или полным
выпадением функции щитовидной железы и изменениями различных органов и систем,
обусловленными недостаточным содержанием в организме тиреоидных гормонов. 1.
Первичного –обусловлен поражением самой щитовидной железы: 2. Вторичного –
обусловлен поражением гипофизарной зоны и снижением секреции тиреотропина: 3.
Третичного – обусловлен поражением гипоталамуса и снижением секреции тиреолиберина:
4. Периферического –избирательная резистентность к Т4 и др.) Этиология - а) врожденного-
наследственные дефекты б) приобретенного: травмы.воспалительные процессы в области
гипофиза. опухоль,лекарственные воздействия . КЛИНИКА - 1. жалобы:- общая и мышечная
слабость, повышенная утомляемость, сонливость, - ощущение постоянной зябкости-
увеличение массы тела, отечность лица, рук- снижение памяти, затруднение речи- запоры-
сухость кожи, выпадение волос- нарушение половых функций- изменение тембра голоса на
низкий, грубый, снижение слуха .2.Объективно при осмотре:- больные адинамичны,
апатичны, заторможены, сонливы, медлительны- речь замедлена, больные с трудом
произносят слова; голос низкий, грубый- кожа бледно-желтоватая .холодная, - волосы
тусклые, ломкие, выпадают на голове, бровях (симптом Хертога), - температура тела
снижена- руки и ноги отечны.Диагностика гипотиреоза:1) OAK: к лейкопения, лимфоцитоз,
увеличение СОЭ.2) ОАМ: уменьшение диуреза, возможна протеинурия.3) БАК: увеличение
содержания в крови холестерина, триглицеридов, пребета- и беталипопротеинов, снижено
содержание общего белка и альбуминов.4) ЭКГ: брадикардия, низкий вольтаж зубцов,
снижение интервала ST книзу от изолинии 5) УЗИ щитовидной железы: уменьшение
размеров при первичном гипотиреозе Лечение - Лечение первичного гипотиреоза -
заместительная терапия тиреоидными препаратами: L-тироксин / эутирокс таб. 50 и 100 мкг;
трийодтиронин таб. 20 и 50 мкг; тиреотом (в 1 таб. 40 мкг Т4 и 10 мкг Т3) и тиреотом форте
(в 1 таб. 120 мкг Т4 и 30 мкг Т3). - заместительная терапия проводится в течение всей жизни

23. эндемический зоб


Эндемический зоб -- заболевание, поражающее население в географических районах,
биосфера которых бедна йодом. Недостаточное поступление йода в организм приводит к
понижению выработки тиреоидных гормонов,. Спорадический зоб возникает у людей,
проживающих вне районов зобной эндемии, вследствие недостаточного всасывания йода в
кишечнике, гормональных нарушений и др. Наблюдается у женщин в 8--10 раз чаще, чем у
мужчин. Клиника. ведущим симптомом эндемического и спорадического зоба является
увеличение щитовидной железы.По функциональному состоянию зоб может быть: а)
гипертиреоидный -- функция щитовидной железы повышена; б) эутиреоидный -- функция
щитовидной железы не нарушена; в) гипотиреоидный -- функция щитовидной железы
снижена. По локализации различают шейный, загрудинный, частично загрудинный,
позадипищеводный зоб и зоб корня языка. У этих больных замедлены реакции на внешние
раздражители, понижена умственная и физическая работоспособность. Больные отмечают
вялость, зябкость. При исследовании выявляется брадикардия, снижение рефлексов.
Симптомы: наиболее часто больные отмечают "чувство неловкости" в области
шеи .Нарушение дыхания -- наиболее частый симптом загрудинного зоба. Нередко (особенно

14
при загрудинном зобе) больные жалуются на состояние тяжести в голове при наклонах тела.
При осмотре этих больных можно отметить расширение вен шеи, характерный рисунок
"голова медузы" в области верхней части грудной стенки. ДИАГНОСТИКА- 1. Общий
анализ крови и мочи без существенных изменений. 2. содержание в крови Тз и Т4 в
пределах нормы либо может отмечаться некоторое возрастание Тз . 3. УЗИ щитовидной
железы. При диффузной форме выявляется диффузное увеличение щитовидной железы
различных степеней, Лечение: при небольших диффузных зобах назначают тиреоидин. При
большом диффузном зобе, вызывающем сдавление трахеи и сосудов, показана резекция
щитовидной железы. При всех формах узлового зоба, если учитывать возможность
малигнизации узлов, необходимо хирургическое вмешательство -- резекция щитовидной
железы с последующим срочным гистологическим исследованием, которое определяет
дальнейшую тактику.

24. Сахарный диабет


Этиология: (аутоимунный ,идиопатический) развивается в следствии деструкции b-клеток
поджелудочной железы,приводящей к абсолютной инсулиновой недостаточности ,
возникает у молодых в возрасте до 20 лет .Клиника: в стадии компенсации жалобы
отсутствуют .Декомпенсации: сухость во рту , жажда, полиурия, увеличение чувства
голода ,плохой сон, повыш.утомляемость,зуд кожи и промежности ,головная боль, снижение
зренияДиагностика ОАК И БАК, общий анализ мочи, выполнить гликемический профиль,
тест на толерантность к глюкозе, пробу Реберга. Рекомендовано УЗИ сердца и почек,
исследование глазного дна,ЭКГ.

25 ожирение
Ожирение- это накопление жира в организме, приводящее к появлению избыточной массы
тела. Этиология :низкая физ. активность, неправильное питание (переедание),снижение
обменных процессов организма, беременность, лактация, климакс, нарушения в эндокринной
системе, стрессовые состояния. Клиника : избыточная масса тела, стрии на теле, грыжи,
эстетические дефекты :второй подбородок. Диагностика : анамнез жизни пациента: наличие
избыточной массы тела у ближайших родственников, особенности профессии, образ жизни и
питания, произвести расчет ИМТ (по Кетле), Оценить жировые отложения из соотношения
объема талии к объему бедер: в норме – до 0,8 у женщин, до единицы – у мужчин.,назначить
лабораторные исследования на наличие гормональных или ферментных нарушений. Лечение
: Схема терапии зависит от причины болезни:1.Алиментарное ожирение лечит диетолог:
составит диету, вместе со специалистом ЛФК подберет оптимальный уровень физической
нагрузки с учетом проблемных зон, назначит препараты, влияющие на метаболизм, аппетит,
всасываемость жира и т. д.2.К лечению психогенного ожирения привлекаются психологи и
психотерапевты, которые проведут специальные тренинги, назначат
антидепрессанты.3.Симптоматическое ожирение может только корректироваться диетой или
упражнениями, единственным методом лечения станет борьба с основным
заболеванием.4.При ожирении III–IV степени применяется бариатрическая хирургия
(резекция или бандаж желудка).

26. Ревматоидный полиартрит


Ревматоидный артрит- это системное воспалительное заболевание соединительной ткани с
преимущественным поражением мелких суставов . Этиология:*генетическая
предрасположенность – риск заболеть ревматоидным артритом увеличен примерно в 4 раза у
кровных родственников больных ревматоидным артритом.*инфекционные агенты*психо-
15
эмоциональный стресс и чрезмерная физическая нагрузка Клиника:- утренняя скованность
в суставах, продолжающаяся не менее 1 часа- артриты суставов кисти- симметричность
поражения суставов- ревматоидные узелки — подкожные уплотнения в области локтей
(встречаются у 10-20% больных).· Со стороны сердечно-сосудистой системы: перикардит,
васкулит, гранулематозное поражение клапанов, атеросклероз· Дыхательная система:
плеврит, интерстициальные заболевания. · Кожа: ревматоидные узелки, утолщение и
гипотрофия, васкулит, сетчатое ливедо. · Нервная система: компрессионная нейропатия,
сенсорно-моторная нейропатия, множественные мононевриты, цервикальный миелит.·.·
Почки: амилоидоз, васкулит, нефрит Кровь: анемия, тромбоцитоз, нейтропения.
Диагностика: анализ крови (общий, клинический); УЗИ; артроскопию; МРТ; пункцию
суставной жидкости; рентгенографию. Лечение:1Назначение нестероидных
противовоспалительных средств (НПВС) внутривенно, внутримышечно или внутрисуставно,
в дополнение могут быть использованы мази; 2Применение глюкокортикостероидов
(гормональных противовоспалительных средств; 3Употребление антидепрессантов при
развитии артрита на фоне стрессов; 4назначение гастропротекторов и ингибиторов
протонной помпы, например, омепразола

II. Ситуационно - клинические задачи с 3 заданиями.


Определите неотложное состояние пациента.
Аргументируйте ответ
Составьте алгоритм оказания догоспитальной помощи .

1.Легочное кровотечение. Кровохаркание- выделение из дыхательных путей при кашле


крови или мокроты с примесью крови (до 5мл в течение суток). Выделение крови из легких
более 5мл за сутки называется легочным кровотечением. Клинические проявления:
Выделение алой пенистой крови при кашлевом толчке Слабость Головокружение Бледность
кожных покровов Снижение АД Учащение пульса слабого наполнения и напряжения
Неотложная медицинская помощь: Придать больному полу сидячее положение Дать
проглотить кусочек льда Ввести: Этамзилата натрия 12,5%- 3мл + 10мл физ-раствора;
Аминокапроновой кислоты 5% - 50мл в/в капельно; Викасол 3мл-в/м.

2. спонтанный пневмоторакс Больному следует придать положение полусидя с опорой для


спины или лежа с поднятым головным концом носилок, удалить лишних присутствующих,
оставив лишь тех, кто необходим; открыть окно, расстегнуть одежду и поясной ремень,
чтобы облегчить дыхательные экскурсии грудной клетки и диафрагмы. Эти простые
мероприятия достаточно эффективны и должны быть выполнены в первую очередь.
Неотложный характер приобретают мероприятия при напряженном клапанном и
двустороннем пневмотораксе, а также при гемопневмоторак-се. Для уменьшения тяжелых
нарушений дыхания и кровообращения стараются снизить внутриплевральное давление. Для
этого плевральную полость пунктируют и удаляют из нее не менее 500 - 600 мл воздуха. Во
время транспортировки необходимо предусмотреть условия, обеспечивающие возможность
свободного дыхания: приподнятое положение или полусидя, свежий воздух, ингаляции
кислорода. При наличии осложнений больного направляют в операционную (перевязочную),
минуя приемное отделение. Следует возможно реже перекладывать больного с одних
носилок на другие. Рентгенологическое исследование грудной клетки проводят сразу же по
поступлении больного в стационар. Боль уменьшают введением морфина, фентанила, других
наркотиков или анальгетиков. Если возможно, выполняют вагосимпатическую блокаду.

16
Кашель подавляют мощными противокашлевыми препаратами (кодеин, тусупрекс, либексин
и др.).

3. Гемоптизия Кровохаркание- выделение из дыхательных путей при кашле крови или


мокроты с примесью крови (до 5мл в течение суток). Выделение крови из легких более 5мл
за сутки называется легочным кровотечением. Клинические проявления: Выделение алой
пенистой крови при кашлевом толчке Слабость Головокружение Бледность кожных
покровов Снижение АД Учащение пульса слабого наполнения и напряжения Неотложная
медицинская помощь: Придать больному полу сидячее положение Дать проглотить кусочек
льда Ввести: Этамзилата натрия 12,5%- 3мл + 10мл физ-раствора; Аминокапроновой
кислоты 5% - 50мл в/в капельно; Викасол 3мл-в/м.

4. Приступ бронхиальной астмы. •Приступ удушья с затрудненным дыханием


•Нарастающая отдышка в покое •Акроцианоз •Повышенная потливость •Жёсткое дыхание
сухими рассеянными хрипами и последующим формированием участков «немого» легкого
•ахикардия •Высокое АД •Участие в дыхание вспомогательной мускулатуры Неотложная
медицинская помощь: 1.Кислородо терапия: непрерывно инсуфлировать через маску
наркозного аппарата или ингалятора кислородно-воздушную смесь с содержанием кислорода
не белее 30%-40%. 2.Инфузионая терапия – направленная на восполнения дефицита ОЦК,
устранение гемоконцентрации и дегидратации по гипертоническому типу; в/в 5%-раствора
глюкозы, реополиглюкина-1000 мл за 1 ч оказания помощи. Кол-во реополиглюкина должно
составлять 30% от общего объема инвазионных растворов. 3.Эуфелин 2,4%-10 мл в/в
медленно в течение 5 минут. 4. Преднизолон 120-180 мг в\в. 5. Гепарин -5000 ЕД действия
в/в капельно с одним из плазмозамещающих растворов.

5 гипертермия. Гипертермический синдром — устойчивое повышение температуры тела.


Помощь: вызвать врача, успокоить, мониторинг, снять с больного теплую одежду, убрать
лишние одеяла, оставить только легкое покрывало или простыню; • приложить холод к
голове и паховым областям, можно на живот и в подмышечные области, к сосудам шеи; • все
тело обильно смочить и растереть спиртом, водкой, 6—9%-ным раствором уксуса. Сразу
после высыхания кожи процедуру повторить несколько раз. При этом желательно избегать
сквозняков; • обильно поить больного негорячими жидкостями; • ввести внутримышечно
литическую смесь: анальгин 50%-ный — 1 —2 мл и димедрол, супрастин, тавегил или
пипольфен — 1 мл. Детям эти препараты вводят из расчета 0,1 мл на год жизни. Допустимо
применение этих препаратов, а также аспирина, парацетамола и т. п. в виде таблеток внутрь,
а также использование ректальных свечей с жаропонижающими веществами. Пипольфен не
назначают пожилым из-за опасности развития психотических реакций; • запрещается делать
компрессы, ставить горчичники, парить ноги и другие тепловые процедуры; • если у
больного исходно отмечается выраженная бледность кожных покровов (спазм
периферических сосудов) или подъем артериального давления, можно ввести дополнительно
спазмолитики (дибазол, папаверин, магнезию).

6. гипертонический криз Успокоить, уложить, дать седативные, дать понижающие АД,


каптоприл 0,25 под язык, если не помогло через 15 минут ещё.

7. Приступ стенокардии Нитроглицерин под язык, не помогло через 2-3 минуты, воздух,
снять одежду, оксигенотерапия, уложить, обезболивающее.

17
8. Острый инфаркт миокарда Нитроглицерин под язык каждые 5 минут, тахикардия
метопролол под язык, обезболивающее, уложить, аспирин.

9. Липотимия Клинические проявления: 1. Кратковременная утрата сознания (на 10–30 с) 2.


В анамнезе отсутствуют указания на заболевания сердечно-сосудистой, дыхательной систем,
ЖКТ, не отягощен акушерско-гинекологический анамнез. Первая медицинская помощь: 1)
придать телу больного горизонтальное положение (без подушки) с немного приподнятыми
ногами; 2) расстегнуть ремень, воротник, пуговицы; 3) опрыскать лицо и грудь холодной
водой; 4) растереть тело сухими руками – руки, ноги, лицо; 5) дать больному вдохнуть пары
нашатырного спирта; 6) внутримышечно или подкожно ввести 1 мл 10 %-ного раствора
кофеина, внутримышечно – 1–2 мл 25 %-ного раствора кордиамина.

10. Коллапс Первая медицинская помощь: 1.уложить больного горизонтально с


приподнятыми нижними конечностями; 2.согреть больного(укрыть больного грелками);
3.ввести 1,0мл 20%р-ра кофеина подкожно; 4.оксигенотерапия(увлажненный кислород); 5.
в/в ввести 90мг преднизолона на физ.растворе; 6.в/в капельно 400,0мл реополиглюкина или
полиглюкина; Провести патогенетическую терапию: при геморрагическом коллапсе
провести остановку кровотечения,ввести кровеостанавливающие средства: -викасол-3,0мл
в/м; -этамзилат натрия 12,5%-2,0мл в/в; -аминокапроновую кислоту 5%-50,0мл в/в капельно;
-хлорид кальция-10,0мл 10%р-рав/в; •При кардиогенном коллапсе-лечение инфаркта
миокарда; •При отравлении ,ожогах ,инфекциях провести дезинтоксикационную терапию

11. Кардиогенный шок кардиогеный шок- наиболее тяжелое осложнение инфаркта


миокарда. 1.Вызвать врача 2.Успокоить, уложить горизонтально, создать физический и
психический покой, опустить головной конец, приподнять ножной.3.Подключить
увлажнённый кислород4.Расстегнуть стесняющую одежду 5.Доступ
воздуха6.Мониторинг(давление, пульс, дыхание, температура7.Госпитализация Если
больной находится в бессознательном состоянии, интубируют трахею;осуществляют
инфузионную терапию раствором реополиглюкина. Кроме того, вводят преднизолон,
антикоагулянты(гепарин вв) и тромболитики(стрептокиназа)производят введение
вазопрессоров(адреналин 0,5 мл вв) В случае фибрилляции желудочков — дефибрилляция;
При асистолии (прекращение сердечной деятельности) — непрямой массаж сердца.

12. Сердечная астма сердечная астма –приступы одышки и удушья, возникающие из-за
застоя крови в легочных венах при нарушении работы левых отделов сердца. 1. вызвать
врача 2. успокоить, усадить удобно с опущенными с кровати ногами 3. расстегнуть
стесняющую одежду 4. доступ воздуха 5. Увлажненный кислород 6. мониторинг(пульс,
давление, чдд) 7. Под язык больному положить одну таблетку Нитроглицерина. 8. По мере
рассасывания Нитроглицерина, дать больному одну таблетку аспирина. Данное лекарство
действует на густоту крови. Разжижая кровь, помогает сердцу лучше функционировать. 9.
Если по истечении десяти минут Нитроглицерин не оказывает ощутимого эффекта,
повторить дозу. В особенно тяжелых случаях можно принимать до пяти таблеток препарата с
интервалом пять-десять минут

13. Отёк лёгкого отёк лёгкого - это патологическое состояние, характеризующееся накоплением
жидкости в легких с последующим нарушением газообмена и дефицитом кислорода в организме.
1.вызвать врача 2. успокоить, усадить удобно в полусидячем положении 3. расстегнуть стесняющую
одежду 4. доступ воздуха 5. подача плевательницы 6. мониторинг(пульс, давление, чдд) 7.

18
кислород+пеногаситель(спирт этиловый) 8. наложить жгуты – один на руку, два на ноги, менять
через каждые 15-20 минут(записать время наложения)9. мочегонные препараты(фурасемид, лазикс)

14. Желудочное кровотечение Неотложная медицинская помощь: Придать больному полу


сидячее положение Дать проглотить кусочек льда Ввести: Этамзилата натрия 12,5%- 3мл +
10мл физ-раствора; Аминокапроновой кислоты 5% - 50мл в/в капельно; Викасол 3мл-в/м.
холод на живот.

15. Желчная колика Первым пунктом плана оказания медицинской помощи должен стать
вызов медиков на место происшествия. Далее придется постараться над тем, чтобы
успокоить пострадавшего. Чтобы немного снизить болезненность, человека укладывают на
ровную поверхность, поворачивая на правый бок. Под него подкладывают грелку с теплой
водой, предварительно обернув ее полотенцем. Чтобы снизить дискомфорт, используют
обезболивающие препараты. Для повышения результативности за основу берутся
инъекционные лекарства. Их вводят внутримышечно.(бралгин, литическая смесь, папаверин
с плетифелином, спазмалгон).

16. Печёночная кома В первую очередь необходимо определить фактор, который


спровоцировал развитие печеночной комы, устранить влияние этого фактора. Инфузионная
терапия – 50-100 мл/кг в сутки, с контролем и коррекцией электролитного баланса, кислотно-
основного состояния. Раствор глюкозы 5-10 г/кг в сутки. Мероприятия, направленные на
уменьшение уровня аммиака в крови: назначение гепатопротекторов, препаратов,
снижающих уровень аммиака в крови, сифонные клизмы и промывание желудка, назначение
антибиотиков для уменьшения количества продукции и всасывания продуктов распада белка
из кишечника. Экстракорпоральная детоксикация (гемосорбция, гемодиализ) по показаниям.
Противосудорожные, седативные препараты по показаниям. Для пациентов с тяжелой
печеночной энцефалопатией, не поддающейся медикаментозному лечению, единственно
эффективным средством остается трансплантация печени.

17. Почечная колика 1.Вызвать врача. 2.Обеспечить больного теплом: его можно поместить
в горячую ванну, но только в сидячем положении. Перед этим необходимо убедиться в том,
что у него отсутствуют противопоказания к горячей ванне (сердечно-сосудистые
заболевания в анамнезе, беременность), в противном случае можно использовать теплую
грелку, которую прикладывают к больной стороне. 3.Больному можно дать спазмолитики –
например, Папаверин или Дротаверин – это будет способствовать расслаблению стенки
мочеточника. Также можно принять комбинированые препараты с спазмолитическим
действием.

18. Уремическая кома 1. Уменьшение интоксикации: промывание желудка и кишечника 4%


р-ром гидрокарбоната натрия, капельные щелочные клизмы 2. Коррекция водно-
электролитных нарушений: в/в инфузии 5% р-ра глюкозы и изотонического р-ра хлористого
натрия (до 2-3 л), при наличии отеков и анурии количество вводимой жидкости
уменьшается, назначается лазикс 40-200 мг и более; для коррекции ацидоза - в/в 200 мл 4%
р-р гидрокарбоната натрия, при гиперкалиемии – в/в капельно 30 мл 10% р-р глюконата
кальция или хлорида кальция, 200-300 мл 40% р-ра глюкозы с добавлением 40-80 ЕД
инсулина 3. Коррекция повышенного АД: 1-2 мл 5% р-ра пентамина в 100-150
физиологического раствора в/в капельно, 10 мл 2,4% р-ра эуфиллина в/в струйно, 4-5 мл 1%
р-ра дибазола в/в струйно (сернокислая магнезия при уремической коме противопоказана из-
за высокого уровня сульфатов в крови) 4. Симптоматическая терапия: при судорожном
19
синдроме – седуксен/ реланиум / диазепам в/м, в случае развития фибринозного перикардита
– преднизолон 20-30 мг/сут внутрь 5. Единственный способ продлить жизнь больного в
стадии уремической комы – программный гемодиализ.

19. Гипогликемическая кома Причины: передозировка инсулина при сахарном диабете,


недостаточное введение углеводов при лечении инсулином, значительная физическая
нагрузка, избыточная продукция эндогенного инсулина.Клиника:1 ст. изменения
настроения,возбужденность/подавленность, головная боль, мышечная слабость,
беспокойства, развивается тахикардия, обостряется чувство голода, а кожа становится
влажной. 2 ст. повышенная потливость, двигательн возбуждение, гиперемия лица, диплопия.
3 ст. Напоминает припадок эпилепсии, кроме того наблюдается расширение зрачков.
Повышенное артериальное давление, тахикардия и потливость усугубляются.4 ст.
гипогликемическая кома: потеря сознания, влажность кожи, нормальная либо немного
увеличена температура тела, нормальное дыхание, повышенность периостальных и
сухожильных рефлексов, расширен-ность зрачков, учащенный пульс и усиленный тон
сердца. Последняя заверша-ющая стадия. снижение тонуса мышц, падает артериальное
давление, обильное потоотделение не наблюдается, наступает арефлексия, сердечный ритм
нару-шен.Ппомощь: 1. Если ребенок в сознании: напоить его сладким чаем с 1-2 кусочками
сахара или 1-2 чайными ложками меда (варенья),г белого хлеба или 50 г печенья. 2. При
внезапной потере сознания: в/в струйно ввестимл 20-40% раствора глюкозы из расчета 2
мл/кг; если ребенок пришел в сознание, его необходимо накормить (манная каша,
картофельное пюре, кисель и др.);при отсутствии эффекта через мин повторить введение 20-
40% р-ра глюкозы в той же дозе, и если эффекта нет — начать в/в капельное введение 10%
раствора глюкозы со скоростью 20 капель в минуту (под контролем гликемии, поддерживая
уровень глюкозы в крови в пределах 6-9 ммоль/л). 3. Если в результате проведенных
мероприятий сознание у ребенка не восстановилось, ввести:раствор глюкагона в дозе 0,5 мл
детям с массой до 20 кг и 1,0 мл — с массой более 20 кг в/м или 0,1% раствор адреналина 0,1
мл/год жизни п/к;3% раствор преднизолона в дозе 1-2 мг/кг вмл 10% раствора глюкозы в/в
капельно (под контролем гликемии). 4. При судорожном синдроме ввести: 0,5% раствор
седуксена в дозе 0,05-0,1 мл/кг (0,3-0,5 мг/кг) в/м или в/в.

20. Гипергликемическая кома Причины: Недостаточное введение инсулина, отказ от


применения, проявление до постановки диагноза, погрешности в питании, травмы,
инфекционные заболевания.Клиника: нарастающая слабость, жажда, потеря аппетита, частое
мочеиспускание, сонливость, гиперемия кожи, учащённое дыхание, потеря сознания ,потеря
кожной чувствительности, судороги, Глаза запавшие. тургор кожи снижен, живот вздут,
снижение АД, запах ацетона изо рта. ПМП:1.Удобное положение.2.Подача
кислорода.3.Ввести инсулин кор дейстрия (актропид). 4.Мониторинг. 5.Измерение глюкозы.

21. Гипогликемическая прекома При легких проявлениях гипогликемии необходимо


срочно съесть около 100 г печенья или несколько кусочков сахара, а еще лучше вместо
сахара - 2-3 ч. ложки меда или варенья. Больному инсулинозависимым диабетом надлежит
всегда иметь при себе небольшое количество сахара или других сладостей. При более
тяжелых проявлениях гипогликемии, то есть при наступлении выраженного прекоматозного
или коматозного состояния, если больной может делать глотательные движения, нужно
влить ему в рот стакан сладкого теплого чая с 3—4 ст. ложками сахара. Перед этим
необходимо, во избежание возможного резкого сжатия челюстей, вставить между
коренными зубами какой-либо фиксатор.
20
22. Гипергликемическая прекома Развивается постепенно, в течение нескольких дней.
Резко усиливается жажда, пропадает аппетит, болит живот. Начинается внезапно, в течение
нескольких минут. Появляется: беспокойство общая слабость; дрожат конечности; чувство
голода; потливость; учащенное сердцебиение; бледнеет лицо. Дают О2, глюкозу и нахрен яд,
чтобы он сдох!!! НЕТ ПРЕКОМЫ ПРИ ГИПЕРГЛИКЕМИИ, НЕ МОГУ Я НАЙТИ ОТВЕТ,
УБЕЙТЕ МЕНЯ, ААААААААААААА

23. Тиреотоксический криз Неотложная помощь Поскольку ТК является критическим,


жизнеугрожающим состоянием, лечебная тактика должна основываться на проведении
реанимационных мероприятий: • быстрое подавление синтеза и высвобождения тиреоидных
гормонов: • назначают внутрь через желудочный зонд пропилтиоурацил (пропицил) в
начальной дозе 600–1200 мг, затем – по 150–250 мг каждые 4 ч или по 300 мг каждые 6 ч,
максимум – до 1500 мг/сут; через 24 ч дозу уменьшают до 100–200 мг каждые 8 ч; – в
отсутствие пропилтиоурацила назначают метимазол (тиамазол, мерказолил); в отличие от
пропилтиоурацила он подавляет синтез тиреоидных гормонов, но не влияет на превращение
Т4 в Т3; тиамазол (мерказолил) также назначают внутрь через желудочный зонд в
измельченном виде в начальной дозе 60 мг, затем – по 30 мг каждые 6–8 ч или по 20 мг
каждые 4 ч (высшая доза – по 60 мг каждые 4 ч), на 2-е сутки – по 10–20 мг каждые 8 ч; если
пероральное введение по каким-либо причинам невозможно, тиреостатики можно вводить в
прямую кишку в виде суппозитория, приготовленного ex tempore из таблеток; введение
тионамидов противопоказано при выраженном нарушении функции печени и
агранулоцитозе; – через 1–2 ч после первого введения тиамазола (мерказолила) или
пропилтиоурацила с целью блокады высвобождения гормонов из щитовидной железы
назначают йодиды (раствор Люголя внутрь по 0,1–0,3 мл через каждые 6 ч или по 0,5 мл
через каждые 8 ч, или 1 г натрия йодида на 1 л 0,9% раствора хлорида натрия внутривенно
капельно 1–2 раза в сутки); считается достаточным введение 100 мг йодида калия 2 раза в
сутки; если у пациента имеется аллергия на йод, его можно заменить литием в дозе по 300 мг
внутрь каждые 8 ч ; введение йодидов без предварительного назначения тиреостатиков
может резко усилить синтез тиреоидных гормонов и ухудшить состояние больного;

24. Гипотиреоидная кома 1. Догоспитальный этап: Вызвать врача. Оказать первую


врачебную помощь. Укутать тело одеялами для уменьшения теплоотдачи. Для ликвидации
гипоксии через носовые катетеры дать увлажненный кислород. Обеспечить доступ к венам,
установить катетер на вену. Если установлена гипотиреоидная кома, тактика медицинской
сестры должна быть четкой, совместная работа с доктором - быстрой, слаженной: Для
диагностики осложнений взять кровь на содержание тироксина, тиреотропина,
трийодтиронина, глюкозы, кортизола, хлоридов, натрия, КЩР, газового состава. Для
контроля диуреза проводится катетеризация мочевого пузыря. Для профилактики аспирации
рвоты вводится в желудок зонд. Для диагностики осложнений — ЭКГ, контроль ЧДД,
температуры, гемодинамики. "Реополиглюкин" внутривенно-капельно 500 мл.
Дезинтоксикация — глюкоза 40 % в/в струйно — 20-30 мл; затем в/в капельно вводится
глюкоза 5 % (500 мл).

25. Острое нарушение мозгового кровообращения Первое, что нужно сделать, если у
больного появились симптомы, позволяющие заподозрить острое нарушение мозгового
кровообращения, - вызвать бригаду скорой медицинской помощи. Больного нельзя
тревожить. Поэтому попросите удалиться тех родственников и близких людей, кто реально
помочь не может.Уложите больного так, чтобы голова и верхняя часть туловища были
21
приподняты, расстегните воротник для облегчения дыхания. Необходимо обеспечить доступ
свежего воздуха, открыть форточку. Если у больного возникла рвота, поверните его голову
набок и очистите полость рта платком или марлей.

26. Острая алкогольная интоксикация. Неотложная помощь: 1.Промывание желудка через


зонд и ускорение элиминации при помощи в\в гипертонического раствора глюкозы с
инсулином (500мл 20% р-ра глюкозы и 20ед инсулина), 500 – 1000мл 4% р-ра
гидрокарбоната натрия; 2.В глубоком коматозном состоянии показано использование метода
фарсированного диуреза. 3.При снижении АД, назначают сердечно-сосудистые средства
(эфедрин, строфантин) 4. Для профилактики аспирационной пневмонии с момента
поступления в стационар больным с нарушением дыхания, назначают антибиотики.

III. Демонстрация практических навыков в соответствии с алгоритмом.

Определение, цель, показания, необходимые материалы, подготовка мед сестры,


подготовка/обучение больного, выполнение техники, реорганизация рабочего места,
документирование, последующий уход за больным.

1.внутрикожная инъекция Цель: Диагностические, аллергические пробы. Показания: 1.


Диагностические тесты. Противопоказания: Аллергия к препарату. Оснащение: 1.
Туберкулиновый шприц ёмкостью 1 мл для в/к инъекций 2. Игла для в/к инъекций с
внутренним диаметром 0,4 мм и длиной 15 мм. 3. Стерильные марлевые салфетки в емкости
со спиртсодержащим антисептиком 4. Ампулы с аллергенами, сыворотками. 10. Лоток
стерильный со стерильной салфеткой (ПР 38/177) 11. Ёмкость для дезинфекции шприцев с
дез. Средством 12. Емкость для дезинфекции ватных шариков с дез. средством 13.
Иглосъемник с дез.средством для дезинфекции игл 5. Пилочки. 6. Пинцет стерильный в
емкости в 6% перекисью водорода 10.Перчатки стерильные одноразовые. Алгоритм
выполнения внутрикожной инъекции: I. Подготовка к процедуре: 1. Представьтесь пациенту,
объясните ход и цель процедуры. Убедитесь в информированном согласии пациента. 2.
Попросите пациента освободить место инъекции (внутренняя поверхность предплечья 3.
Наденьте маску. 4. Определите непосредственно место инъекции 5. Обработайте руки
гигиеническим способом 6. Наденьте стерильные одноразовые перчатки 7. Наберите
лекарственный препарат согласно набору лекарственных средств из ампулы и флакона. 8.
Обработайте стерильные одноразовые перчатки спиртсодержащим антисептиком II.
Выполнение процедуры: 9. Обработайте антисептиком место инъекции последовательно
двумя марлевыми салфетками: сначала большую площадь, а затем меньшую. Дождитесь,
пока высохнет антисептик. 10. Возьмите шприц с иглой в правую руку. 11. Обхватив левой
рукой предплечье пациента снаружи фиксируйте кожу (не растягивайте). 12. Возьмите
шприц в правую руку: 2-й палец придерживает иглу за канюлю, а 1-й, 3-й, 4-й и 5-й пальцы
фиксируют шприц за цилиндр. 13. Введите иглу срезом вверх параллельно коже пациента
(под углом 15 градусов) на самый кончик иглы. 14. Приподнимите кожу срезом иглы в виде
«палатки». 15. В таком положении иглы, отпустите левую руку, перенесите ее на поршень
шприца и, надавив на поршень, введите назначенную дозу лекарственного средства. 11.
Извлеките быстрым движением иглу (стерильный шарик не прикладывайте). 12. Спросите у
пациента о его самочувствии,. III. Окончание процедуры: 13 . Подвергнуть дезинфекции весь
использованный материал . Для этого из контейнера «Для дезинфекции шприцев», через
иглу, в шприц набрать дез.средство, снять иглу с помощью иглосъемника, шприц поместить
в соответствующий контейнер. Шарики поместить в контейнер «Для использованных
22
шариков». Подвергнуть дезинфекции лотки. 14 . Снять перчатки, поместить их в
непромокаемый пакет соответствующего цвета для последующей утилизации. 15 .
Обработать руки гигиеническим способом, осушить 16. Сделать соответствующую запись о
результатах выполнения в листе наблюдений сестринской истории болезни, Журнале
процедурной м/с.

2.подкожная инъекция Цель: Лечебная. Показания: 1. Введение небольших объёмов


лекарственных веществ по назначению врача. 2. Введение масляных растворов. 3. Введение
инсулина. 4. Введение гепарина. 5. Выполнение проф. прививок. 6. Введение лекарственных
веществ, быстро всасывающихся в рыхлой подкожной клетчатке. Противопоказания: 1.
Аллергия к препарату. 2. Инфильтраты в местах инъекции. 3. Отёчная подкожно-жировая
клетчатка. Оснащение: 1. Стерильный лоток, со стерильной салфеткой 2. Нестерильный
лоток 3. Перчатки стерильные одноразовые 4. Маска 5. Шприц стерильный 1- 2 мл. 6. Иглы,
длиной 20 мм. 7. Ампулы с лекарственным веществом. 8. Стерильный пинцет в емкости с 6%
перекисью водорода 9. Марлевые салфетки со спиртсодержащим антисептиком 3-5 шт для
обработки инъекционного поля 10. Спиртсодержащий антисептик для гигиенической
обработки рук 11. Ёмкость для дезинфекции шприцев с дез. средством 12. Емкость для
дезинфекции ватных шариков с дез. средством 13. Иглосъемник с дез.средством для
дезинфекции игл . Алгоритм выполнения подкожной инъекции: I. Подготовка к процедуре:
1. Представиться пациенту, объяснить ход и цель процедуры. Убедиться в наличии у
пациента информированного согласия на предстоящую процедуру и отсутствие аллергии на
данное лекарство. 2. Сообщите пациенту необходимую информацию о лекарственном
препарате. 3. Помогите пациенту занять нужное положение. 4. Наденьте маску. 5.
Обработайте руки гигиеническим способом спиртсодержащим антисептиком 6. Оденьте
стерильные перчатки. 7. Наберите в шприц назначенное лекарственное средство, согласно
алгоритму выполнения набора лекарств из ампулы 8. Определите место инъекции. 9.
Обработайте стерильные одноразовые перчатки спиртсодержащим антисептиком II.
Выполнение процедуры: 10. Обработайте место инъекции (большую площадь) стерильными
марлевыми салфетками, смоченным спиртсодержащим антисептиком 11. Обработайте место
инъекции (меньшую площадь) второй стерильной марлевой салфеткой с антисептиком.
Третью марлевую салфетку зажмите между 4-м и 5-м пальцами левой руки. 12. Грязные
марлевые салфетки сбрасывать в нестерильный лоток 13. Выпустите воздух из шприца в
колпачок (шприц держите перпендикулярно полу, 2-й палец придерживает иглу за канюлю).
14. Возьмите шприц в правую руку, 2-й палец на канюле иглы, остальные – на цилиндре
шприца. 15. Возьмите кожу в месте инъекции в складку первым и вторым пальцами левой
руки так, чтобы получился треугольник, обращенный основанием вниз. 16. Введите иглу под
кожу в основание треугольника под углом 45 градусов к поверхности кожи, срезом иглы
вверх на 2/3 её длины. 17. Отпустите левую руку и перенесите ее на поршень шприца 18.
Левой рукой медленно введите лекарственный препарат. 19. Левой рукой прижмите
стерильную марлевую салфетку к месту инъекции 20. Правой рукой быстрым движением
извлеките иглу. 21. Спросите пациента о самочувствии. III. Окончание процедуры: 22.
Подвергнуть дезинфекции весь использованный материал. Для этого из контейнера «Для
дезинфекции шприцев», через иглу, в шприц набрать дез.средство, снять иглу с помощью
иглосъемника, шприц поместить в соответствующий контейнер. Марлевые салфетки
поместить в контейнер «Для использованных салфеток». Подвергнуть дезинфекции лотки.
23. Снять перчатки, поместить их в непромокаемый пакет соответствующего цвета для
последующей утилизации 24. Обработать руки гигиеническим способом, осушить 25.
23
Сделать соответствующую запись о результатах выполнения в листе наблюдений
сестринской истории болезни, Журнале процедурной м/с.

3. Внутримышечная инъекция Цель:лечебная и профилактическая. Показания: - для


достижения более быстрого терапевтического эффекта; - при невозможности перорального
применения препаратов; - использование препаратов применяемых только для внутри-
мышечных введений; - плохое рассасывание препаратов при подкожном введении.
Противопоказания: - индивидуальная непереносимость препарата; - использование
препаратов только для внутривенных инъекций; - уплотнение (инфильтрат) в мышцах после
предыдущих инъекций; - воспалительные изменения кожи в месте инъекции; - нарушение
целостности кожи в месте инъекции. Оснащение: манипуляционный столик, кушетка, шприц
однократного применения 5-10 мл., длина иглы 40 - 60мм.(1 шт.), шприц однократного
применения 2-5 мл., длина иглы 20-25 см.(1 шт.), игла для набора, лоток почкообразный
стерильный (1 шт.), стерильная пеленка, лоток нестерильный-1 шт., непрокалываемый
контейнер и пакет для использованных шприцев - 1 шт, дезинфицирующее средства,
антисептик для обработки инъекционного поля, для обработки рук, марлевые шарики или
салфетки (3 шт.), мыло жидкое, перчатки стерильные (1 пара). Алгоритм манипуляций
Этапы Обоснование I. Подготовка к процедуре. 1.1. Подготовить все необходимое к
манипуляции. Проверить фамилию больного, соответствие препарата врачебному
назначению, прозрачность, цвет, срок годности. Уточнить аллергологический анамнез.
Эффективность проведения манипуляции. Предупреждение осложнений 1.2. Разъяснить
пациенту смысл манипуляции. 1.3. Помогите пациенту занять удобное положение (если
требуется, отгородите ширмой). 1.4. Провести гигиеническую антисептику рук. 1.5. Надеть
перчатки (стерильные), непосредственно перед проведением инъекции целесообразно
обработать нестерильные перчатки раствором антисептика. 1.6. Обработать шейку ампулы
(крышку флакона) шариками со спиртом – двухкратно. 1.7. Подготовить шприц и иглу для
набора препарата. 1.8. Набрать лекарство в шприц из ампулы или флакона. 1.9. Произвести
смену иглы (надеть на конус шприца иглу для в/м инъекции). Использованную иглу
поместить в емкость с дез. раствором. II. Выполнение процедуры. 2.1. Предложить пациенту
лечь (или уложить пациента на живот или на бок), освободить место для инъекции. 2.2.
Определите место инъекции (выбрать, осмотреть, пропальпировать). 2.3. Обработайте место
инъекции в одном направлении раствором антисептика, первым шариком - широкое поле,
площадь которого примерно 4х6 см., вторым – непосредственно место инъекции, делая
мазки в одном направлении, подождите, пока антисептик испарится (место инъекции должно
быть сухим). 2.4. Туго натянуть кожу большим и указательным пальцами одной руки. Взять
шприц другой рукой, придерживая канюлю иглы пальцем. 2.5. Введите иглу быстрым
движением под углом 90º на 2/3 ее длины, оставляя 1см. над поверхностью кожи(мизинец
должен быть на канюле иглы, 2-ой палец на поршне сбоку, а 1-ый, 3-ий, 4-ый-на цилиндре).
Потяните поршень на себя (при введении любого лекарственного средства), убедитесь, что в
цилиндре нет крови. 2.6. Медленно введите лекарственное средство, нажимая на поршень1-
ым пальцем левой руки 2.7. Быстро извлечь иглу, продолжая придерживать ее за канюлю.
Прижать шарик к месту инъекции. 2.8. Сделать легкий массаж места инъекции, не отнимая
шарика от кожи. III .Окончание процедуры. 3.1. Весь использованный инструментарий и
материал подлежат дезинфекции. 3.2. Провести гигиеническую антисептику рук. 3.3.
Сделать соответствующую запись о результатах выполнения в медицинскую документацию.

24
4.внутривенная инъекция I. Подготовка к процедуре: 1. Представиться пациенту,
объяснить ход и цель процедуры. Убедиться в наличии у пациента информированного
согласия на предстоящую процедуру введения лекарственного препарата и отсутствие
аллергии на данное лекарственное средство. 2. Надеть маску. 3. Обработать руки
спиртсодержащим антисептиком, гигиеническим способом согласно требованиям 4. Одеть
стерильные одноразовые перчатки 5. Набрать лекарственный препарат. 6. Положить в
стерильный лоток со шприцем при помощи стерильного пинцета 3 стерильных салфетки
(для обработки места инъекции). II. Выполнение процедуры: 7. Усадить или уложить
пациента так, чтобы рабочая рука была в разогнутом состоянии ладонью вверх. 8. Положить
под локоть клеёнчатую подушечку 9. Осмотреть/ пропальпировать область предполагаемой
венепункции для избежания возможных осложнений. 10. Наложить жгут на одежду пациента
или на полотенце, и попросить пациента несколько раз сжать кисть в кулак, разжать её.
Сжать кисть в кулак. 11. Обработать стерильные одноразовые перчатки спиртсодержащим
антисептиком согласно требованиям 12. Обработать область венепункции последовательно 2
салфетками с кожным антисептиком, движениями в одном направлении от области сверху
вниз, одновременно определяя наиболее наполненную вену. Салфетки после использования
сбрасывать в нестерильный лоток. 13. В левой руке между 4-м и 5-м пальцами зажать
стерильную марлевую салфетку с антисептиком 14. Взять шприц правой рукой, фиксируя
указательным пальцем канюлю иглы. Остальные пальцы равномерно распределены по
цилиндру шприца. 15. Большим пальцем левой руки фиксировать вену в области
венепункции ниже месте инъекции. 16. Правой рукой взять шприц, держа иглу срезом вверх,
параллельно коже, проколоть кожу, затем пунктировать вену (не более чем ½ иглы). При
попадании иглы в вену, ощущается «попадание в пустоту». 17. Убедиться, что игла в вене:
левой рукой потянуть поршень на себя. Появление крови в шприце означает, что игла – в
вене. 18. Левой рукой развязать/ослабить жгут и попросить пациента разжать кулак. Для
контроля иглы в вене ещё раз потянуть поршень на себя, т.к. в момент ослабления жгута игла
может выйти из вены. 19. Левой рукой нажать на поршень, не меняя положения шприца, и
медленно ввести лекарственный препарат, оставив в шприце 0,3-0,5 мл раствора
(профилактика попадания в вену пузырьков воздуха). 20. Левой рукой прижать к месту
инъекции салфетку с кожным антисептиком. Извлечь иглу, попросить пациента согнуть руку
в локте и держать салфетку/ватный шарик у места инъекции 5-7 минут. 21. Убедиться, что
наружного кровотечения в области венепункции нет. III. Окончание процедуры: 22.
Подвергнуть дезинфекции весь использованный материал. Для этого из контейнера «Для
дезинфекции шприцев», через иглу, в шприц набрать дез.средство, снять иглу с помощью
иглосъемника, шприц поместить в соответствующий контейнер. Салфетки поместить в
контейнер «Для использованных салфеток». Подвергнуть дезинфекции лотки. 23. Снять
перчатки, поместить их в непромокаемый пакет соответствующего цвета для последующей
утилизации 24. Обработать руки гигиеническим способом, осушить 25. Сделать
соответствующую запись о результатах выполнения в листе наблюдений сестринской
истории болезни, Журнале процедурной мед.сестры.

5.внутривенное капельное вливание I. Подготовка к процедуре: 1. Представиться


пациенту, объяснить ход и цель процедуры. Убедиться в наличии у пациента
информированного согласия на предстоящую процедуру введения лекарственного препарата
и отсутствие аллергии на данное лекарственное средство, если пациент(ка) в сознании и
он(а) старше 15 лет; в других случаях (дети до 15 лет, без сознания и т.д.) - у законных
представителей. В случае их отсутствия процедура, выполняется без получения
25
информированного согласия. 2. Предложить пациенту опорожнить мочевой пузырь,
учитывая длительность выполнения. 3. Предложить/помочь занять пациенту удобное
положение, которое зависит от его состояния. 4. Одеть маску 5. Обработать руки
гигиеническим способом, осушить 6. Надеть стерильные одноразовые перчатки. 7.
Заполнить устройство для вливания инфузионных растворов однократного применения и
поместить его на штативе для инфузионных вливаний. Заполнение устройства для вливаний
инфузионных растворов однократного применения: · Проверить срок годности устройства и
герметичность пакета. · Прочитать надпись на флаконе: название, срок годности. Убедиться
в его пригодности (цвет, прозрачность, осадок). · Нестерильным пинцетом вскрыть
центральную часть металлической крышки флакона, обработать резиновую пробку флакона
салфеткой, смоченной антисептическим раствором. Ввести во флакон, набранный в шприц
лекарственный препарат. · Вскрыть упаковочный пакет и извлечь устройство для в/в
капельного введения (все действия производятся на рабочем столе). · Закрыть винтовой
зажим, а, ввести иглу жестко соединенную с капельницей, на коротком конце устройства, до
упора в пробку флакона. · Перевернуть флакон и закрепить его на штативе. · Одновременно
с двух сторон сдавить капельницу пальцами и заполнить ее на ½, открыть заглушку
капельницы. · Открыть винтовой зажим и медленно заполнить систему до полного
вытеснения воздуха и появления капель из иглы системы в защитном колпачке иглы. ·
Убедиться в отсутствии пузырьков воздуха в длинной трубке устройства (устройство
заполнено). · Закрыть зажим (в системе воздуха быть не должно!) · Зафиксировать
заполненную систему на штативе. · Положить в стерильный лоток при помощи стерильного
пинцета салфетки/ватные шарики с кожным спиртсодержащим антисептиком, стерильную
салфетку. Приготовить полоску узкого лейкопластыря, шириной 1 см, длиной 4-5 см,
зафиксировать ее на штативе. 7. Доставить в палату манипуляционный столик или лоток, с
размещённым на нём необходимым оснащением, штатив с приготовленной капельницей. 8.
Обследовать/пропальпировать место предполагаемой венепункции для избежания
возможных осложнений. При выполнении венепункции в область локтевой ямки -
предложить пациенту максимально разогнуть руку в локтевом суставе, для чего подложить
под локоть пациента клеёнчатую подушечку. 9. Наложить венозный жгут (на рубашку или
пелёнку) в средней трети плеча так и попросить пациента несколько раз сжать кисть в кулак
и разжать её. 10.Обработать стерильные перчатки спиртсодержащим антисептиком согласно
требованиям II. Выполнение процедуры: 12. Обработать область венепункции
последовательно 2 салфетками с кожным антисептиком, движениями в одном направлении,
одновременно определяя наиболее наполненную вену. 13. Большим пальцем левой руки
фиксировать вену ниже места инъекции. 14. Правой рукой пунктировать вену иглой с
подсоединённой к ней системой (держать иглу срезом вверх); Появление крови в канюле
иглы – означает, что игла в вене. Закрепить иглу и систему лейкопластырем. 15. Попросить
пациента разжать кисть, одновременно левой рукой развязать/ослабить жгут. 16. Открыть
винтовой зажим, отрегулировать винтовым зажимом скорость поступления капель (40-60 в
минуту), вводить лекарственный препарат. 17. Прикрыть иглу стерильной салфеткой,
смоченной антисептиком. 18. Снять перчатки, поместить их в непромокаемый пакет 19.
Обработать руки спиртсодержащим антисептиком, гигиеническим способом согласно
требованиям 20. Наблюдать за состоянием пациента, его самочувствием на протяжении всей
процедуры (в условиях оказания помощи в процессе транспортировки, продолжительность
наблюдения определяется продолжительностью транспортировки), закончить введение
препарата. III. Окончание процедуры: 21. Обработать руки спиртсодержащим антисептиком,
гигиеническим способом согласно требованиям 22. Надеть стерильные перчатки 23.
26
Закрыть винтовой зажим. Приложить к месту пункции стерильный ватный шарик с
антисептиком, извлечь иглу из вены. Попросить пациента согнуть руку в локтевом суставе
на 5-7 минут для полной остановки кровотечения. 24. Убедиться, что наружного
кровотечения в области венепункции нет. 25. Подвергнуть дезинфекции весь
использованный материал. Иглы поместить в иглосъемник, капельницы в емкость для
дезинфекции капельниц.. Салфетки поместить в контейнер «Для использованных
салфеток».Подвергнуть дезинфекции лотки. 26. Снять перчатки, поместить их в
непромокаемый пакет соответствующего цвета для последующей утилизации 27. Обработать
руки гигиеническим способом, осушить 28. Сделать соответствующую запись о результатах
выполнения в листе наблюдений сестринской истории болезни, Журнале процедурной
мед.сестры.

6.термометрия Показания: контроль за процессами терморегуляции у ребенка, перед


некоторыми медицинскими манипуляциями. Противопоказания: воспалительные процессы
на месте измерения температуры, влажная поверхность кожи. Материальное оснащение: -
максимальный медицинский термометр; - термометр «Термотест» (полимерная пластинка
покрытая эмульсией из жидких кристаллов); - 1% раствор хлорамина. Сбор данных -
осмотреть кожу подмышечных впадин. При воспалительных процессах в подмышечных
впадинах осмотреть кожу в паховых складках; - убедиться в отсутствии влажности кожных
покровов; - убедиться в целостности термометра; внимательно оценить назначения врача
(возможно измерение температуры каждые 2 часа) Тактика медсестры - при невозможности
измерить температуру в подмышечной впадине или паховой складке можно измерить
температуру в полости прямой кишки. Алгоритм действия 1) взять обработанный и
высушенный термометр и стряхнуть показания ртутного столба ниже 35°С; 2) протереть
насухо подмышечную впадину или паховую область; 3) поместить резервуар термометра в
подмышечную впадину; 4) прижать плечо к грудной клетке, чтоб резервуар полностью
соприкасался с кожей. Детей грудного возраста запеленать, а более старших усадить на
колени и прижать плечо ребенка к грудной клетке 5) извлечь термометр через 10 мин. и
определить показания ртутного столба. Примечание: при измерении в прямой кишке
термометр смазать вазелином и ввести в анус за внутренний сфинктер; медсестре не
отходить от ребенка в течение всей манипуляции. Оценка - если температура измерялась в
прямой кишке, температура будет на 0,5-0,8°С выше показателей измерения в подмышечной
впадине. Завершение манипуляции - записать показатели в общий температурный лист,
затем перенести эти показатели в температурный лист истории; - термометр погрузить в 1%
раствор хлорамина на 15 мин., затем вынуть, промыть проточной водой и высушить; -
хранить термометр в лотке, на дне которого вата или марля.

7.определение ЧДД, АД, пульса Артериальное давление — давление, которое кровь


оказывает на стенки кровеносных сосудов. Цель: Определить показатели артериального
давления н оценить результаты исследования. Оснащение: Тонометр, фонендоскоп, ручка с
синей пастой, температурный лист, 70% спирт, ватные шарики, бумага. Измерение АД:
Подготовка к процедуре: 1. Предупредить пациента о предстоящей процедуре за 15 минут до
ее начала; 2. Придать пациенту удобное положение сидя или лежа. Выполнение процедуры:
1. Уложить руку пациента в разогнутом положении ладонью вверх, подложив валик под
локоть или попросить пациента подложить под локоть сжатый кулак кисти свободной руки;
2. Выбрать правильный размер манжетки; 3. Наложить манжетку тонометра трубками вниз
на обнаженное плечо пациента на 2-3 см. Выше локтевого сгиба так, чтобы между ними

27
проходил только один палец; 4. Соединить манометр с манжеткой, укрепив его на ней; 5.
Проверить положение стрелки манометра относительно отметки «0» на шкале манометра; 6.
Определить пальцами пульсацию в локтевой ямке, приложить на это место мембрану
фонендоскопа; 7. Закрыть вентиль «груши», нагнетать воздух в манжетку до исчезновения
пульсации в локтевой артерии +20-30 мм. рт. ст. (т.е. несколько выше предполагаемого АД);
8. Открыть вентиль, медленно выпускать воздух, выслушивая тоны, следить за показаниями
манометра; 9. Отметить цифру появления первого удара пупьсовой волны, соответствующую
систолическому АД (АДс); 10. «Отметить» исчезновение тонов, что соответствует
диастолическому АД (АДд); 11. Выпустить весь воздух из манжетки. Окончание процедуры:
1. Снять манжетку; 2. Уложить манометр в чехол; 3. Продезинфицировать головку
фонендоскопа методом двукратного протирания 70% спиртом; 4. Оценить результат высоты
АД и пульсового давления; 5. Сообщить пациенту результат измерения; 6.Провести
регистрацию результата в виде дроби (в числителе - систолическое давление, в знаменателе -
диастолическое). Оценка: в норме артериальное давление зависит от возраста, условий
внешней среды, нервного и физического напряжения. Артериальное давление делится на:
оптимальное — 120 на 80 мм рт. ст., нормальное — 130 на 85 мм рт. ст., высокое, но все же
нормальное — от 135-139 мм рт. ст., на 85-89 мм рт. ст. Высоким считается давление 140 на
90 мм рт. ст. и более. Измерение пульса на лучевой артерии: Подготовка к процедуре:
Придать пациенту удобное положение сидя или лежа. Предложить расслабить руки, при
этом кисти и предплечья не должны быть на весу. Выполнение процедуры: Рука
исследуемого должна находиться в удобном полусогнутом положении, исключающем
напряжение мышц. Исследующий кладет 2-й, 3-й, 4-й пальцы на внутреннюю поверхность
нижней части предплечья в области лучевой кости, располагая большой палец на наружной
поверхности кисти руки; Сила воздействия на артерию, должна быть минимальной, не стоит
придавливать слишком сильно. Найдя пульс, определите его частоту, ритм, наполнение и
напряжение. Время измерения составляет от 30-60 секунд, лучше полную минуту, тогда
удара сердца будет наиболее точным. Если все же вы торопитесь, то измерив, ритмичность
удара за 30 секунд, умножьте этот показатель на два. Если пульс на лучевой артерии
исследовать не удается (при травмах, ожогах), то его определяют на сонной, бедренной,
височной артериях. Окончание процедуры:1. Провести регистрацию качеств пульса в
температурном листе графическим, а в листе наблюдения цифровым способом. 2. Сообщить
пациенту результаты исследования

8.сбор мокроты на микробактерии туберкулёза Цель. Выделение микобактерии


туберкулеза Показания. Подозрение на туберкулез легких. Оснащение. Стерильная сухая
банка с плотно закрывающейся крышкой. Техника взятия мокроты на микобактерии
туберкулеза 1. Накануне вечером пациента предупреждают о предстоящем исследовании
следующим образом: «Завтра с 6.00 утра вам нужно начать собирать мокроту на
исследование. Мокрота на назначенное вам исследование собирается в течение суток. Это
значит, что всю мокроту, которая будет у вас выделяться при кашле, необходимо сплевывать
в эту банку. Банку, пожалуйста, ставьте в прохладное место и плотно закрывайте крышкой».
Необходимо показать пациенту то место, где в течение суток будет храниться банка с
мокротой. 2. Собранную мокроту отправляют в бактериологическую лабораторию. 3.
Результат исследования вклеивают в медицинскую карту стационарного больного.

28
9.катетеризация мочевого пузыря у женщины катетером фолея Цель:лечебная и
диагностическая. Показания:острая и хроническая задержка мочи, недержание мочи,
промывание мочевого пузыря, послеоперационный период, последовый период родов.
Оснащение: 1. Стерильный катетер Фолея. 2. Стерильные перчатки. 3. Стерильные салфетки.
4. Стерильный глицерин. 5. Шприц с изотоническим раствором (объем раствора,
соответствует емкости баллона на катетере Фолея). 6. Впитывающая пеленка. 7. Стерильный
антисептический раствор (например: раствор калия перманганата 1:5000, раствор
фурацилина 0,02%, раствор риванола 0,02%). 8. Стерильный лоток. 9. 2 чистых лотка. 10.
Полоски лейкопластыря. 11. Мочеприемник. Судно. Чистые перчатки. Кувшин с
антисептическим раствором (t – 38 0С). Стерильный лоток с ватно-марлевыми тампонами.
Стерильный корнцанг. Емкость для использованного материала. Впитывающая пеленка.
Обязательное условие: процедура введения катетера проводится при строгом соблюдении
асептики, так как слизистая оболочка мочевого пузыря обладает слабой сопротивляемостью
к инфекции, ежедневное промывание мочевого пузыря, наличие закрытой герметичной
системы для сбора мочи. Подготовка к процедуре 1. Собрать информацию о пациентке.
Доброжелательно и уважительно представиться. Уточнить. как к ней обращаться. Объяснить
суть и ход предстоящей процедуры, если она с нею не знакома. Получить согласие на
проведение процедуры (если пациентка в сознании). Психологическая подготовка пациента к
процедуре. Мотивация пациента к сотрудничеству. Соблюдение права пациента на
информацию. 2. Подготовить оснащение для гигиенической обработки промежности. 3.
Вымыть руки (гигиенический уровень). Надеть перчатки. 4. Помочь пациентке занять
необходимое для процедуры положение: на спине с полусогнутыми разведёнными ногами
(поза «лягушачья нога»). Примечание: для женщин, которые не способны развести бёдра, их
сгибание обеспечивает достаточно хороший доступ к уретре. 5. Подстелить под таз
пациентки впитывающую пеленку; подать судно. Поставить между ногами пациентки
предметы, необходимые для гигиенической обработки наружных половых
органов.Произвести гигиеническую обработку наружных половых органов и промежности.
Убрать судно. 6. Снять перчатки и сбросить их в непромокаемый мешок. 7. Вымыть руки на
гигиеническом уровне и приготовить всё необходимое оснащение для катетеризации.
Выполнение процедуры 8. Вымыть руки на гигиеническом уровне. Надеть стерильные
перчатки. 9. Обложить стерильными салфетками вход во влагалище. 10. Развести в стороны
левой рукой (если вы правша) малые половые губы. Правой рукой взять малую салфетку,
смоченную раствором антисептика, и обработать наружное отверстие мочеиспускательного
канала движениями сверху вниз. Поместить салфетку в лоток для использованного
инструментария. При необходимости повторить процедуру. 11. Попросить ассистента
вскрыть упаковку с катетером. Извлечь катетер из упаковки: держать его на расстоянии 5 – 6
см от бокового отверстия 1 и 2 пальцами, наружный конец катетера держать между 4 и 5
пальцами. 12. Попросить ассистента обильно обработать катетер глицерином (методом
полива над стерильным лотком) или специальной желеобразной смазкой. 13. Ввести катетер
в отверстие уретры до появления мочи, а потом еще на 5 см, вывести мочу в лоток.
Примечание: если вход в уретру хорошо не определяется, требуется консультация врача. 14.
Зафиксировать катетер, наполнив физиологическим раствором надувную манжету. Слегка
потянуть катетер кнаружи, убедиться в том, что он зафиксирован. 15. Соединить наружный
конец катетера с мочеприемником (лучше градуированным). Убедиться, что трубки не
перегибаются, Прикрепить трубку катетера Фолея пластырем к бедру, не натягивая катетер.
16. Следить, чтобы система катетер - мочеприемник была замкнутой. Не промывать! В
случае непроходимости заменить на новый - стерильный. 17. Зафиксировать мочеприемник к
29
кровати ниже её плоскости. 18. Помочь пациентке занять удобное положение. Накрыть её.
Удостовериться, что она чувствует себя нормально. Убрать ширму, если она использовалась.
19. Опорожнять мочеприемник, не нарушая целостности системы, через отводной порт.
20.Дважды в сутки осматривать область уретры вокруг катетера и область промежности;
обрабатывать промежность и 10 см проксимального участка катетера водой с жидким
мылом, осушать одноразовым полотенцем. Окончание процедуры 21.Удалять катетер по
истечении назначенного времени после удаления физиологического раствора из надувной
манжеты. 22.Дезинфицировать весь использованный инструментарий. Утилизировать
одноразовый материал после дезинфекции. 23.Убрать пеленку. Снять перчатки. Вымыть
руки. 24.Записать реакцию пациентки на процедуру в «медицинской карте».

10.применение аэрозольного ингалятора Ингаляцию сегодня называют самым


эффективным способом лечения заболеваний дыхательных путей. В сравнении с пилюлями и
сиропами, этот метод имеет очевидное превосходство. Во-первых, распыленное лекарство
соприкасается почти со всей поверхностью слизистой, быстрее всасывается в кровь и, как
следствие, дает лучший результат и скорейшее выздоровление. Во-вторых, воздействие
лекарственных веществ идет прямо на дыхательные пути, минуя долгий путь приема
препарата через желудок. В-третьих, с помощью ингаляции из дыхательной системы быстрее
выводятся микробы, что облегчает удаление слизи и мокроты. В связи с этим большое
признание среди врачей и пациентов получил небулайзер. Этот прибор преобразовывает
лекарства в аэрозоль и подает его в дыхательные пути больного. Благодаря этому
устройству, сегодня ингаляционная терапия доступна не только в физиокабинетах
медицинских учреждений – пользоваться небулайзером легко и удобно и в домашних
условиях. Пациенты всех возрастов могут применять ингаляцию для профилактики
приступов бронхиальной астмы, для облегчения дыхания и улучшения скорости вдоха, для
ускорения отхождения мокроты во время острых респираторных заболеваний. Наряду с
другими видами ингаляций именно небулайзерная является наиболее безопасной,
комфортной и современной. Прибор обеспечивает непрерывную подачу лекарства, в связи с
чем отсутствует необходимость выполнения глубоких вдохов-выдохов. Еще одно из
наиболее важных преимуществ небулайзеров – это то, что в них не используются
пропелленты – вещества, с помощью которых создается давление, обеспечивающее
распыление. Алгоритм использования небулайзера: Перед началом ингаляции обязательно
вымойте руки с мылом. 1. Соберите прибор. Соедините все трубки, проверьте воздушный
фильтр. 2. Подготовьте необходимый препарат. Для ингаляций лучше использовать
препараты в специальных небулах для каждого применения. Если препарат надо развести,
используйте только 0,9 % раствор хлорида натрия! Никакой другой растворитель нельзя
использовать, а тем более самодельный (минеральная, дистиллированная вода). Препарат из
ампулы набирается стерильным шприцом, затем разводится до объёма 4 мл
физиологическим раствором, выливается в стаканчик. 3. Стаканчик с готовым лекарством
присоединяется к трубке прибора. Сверху надевается лицевая маска по размеру (взрослая
или детская). Взрослому предпочтительнее использовать мундштук, т. к. потери
лекарственной взвеси будут меньше. Маленького ребёнка лучше держать на руках,
приложив маску к лицу, развлекать его, чтобы он не испугался процедуры. 4. Начинайте
ингалироваться. Для этого включите аппарат, держите стаканчик строго вертикально, чтобы
не разлить лекарство. Ингаляция в среднем занимает минут 10. Сигналом для прекращения
ингаляции послужит отсутствие пара из стаканчика (это означает, что просто закончилось
лекарство). 5. После каждой ингаляции необходимо стаканчик, маску и мундштук
30
прополоскать кипячёной водой и высушить. Если проводилась ингаляция с гормональными
препаратами, то кипячёной водой необходимо прополоскать и ротовую полость. 6.
Ингаляцию необходимо проводить спустя 1,5 часа после еды, т. к. в некоторых случаях
возможна рвота, особенно на щелочные растворы. 7. Применять необходимо только те
лекарства, которые назначены врачом, и в соответствующих дозировках. Не занимайтесь
самолечением! 8. Сразу после ингаляции нельзя курить и принимать пищу, лучше всё
отложить на час. 9. Если у вас появились неприятные ощущения, лучше прекратить
ингаляцию на некоторое время, а потом продолжить.

11.пикфлоуметрия это метод определения пиковой объемной скорости форсированного


выдоха Алгоритм действий такой: за 2 часа до исследования нельзя есть. Нельзя курить и
пить алкогольные напитки за 24 часа до пневмотахометрии. За 30 минут перед
исследованием отдохните, посидите в спокойной обстановке, одежда должна быть свободна
в области грудной клетки. Не принимайте бронхолитики за 5 часов до исследования, а если
вы пользуетесь ингалятором — возьмите его с собой. Имейте при себе носовой платок. Для
этой процедуры нет ограничений, ее можно проводить как взрослым, так и детям. Извлечь
устройство из упаковки. При повторных измерениях все части пикфлоуметра должны быть
чистыми Присоединить мундштук пикфлоуметра, установить индикаторный указатель на
ноль Процедуру выполняют стоя или сидя, при этом необходимо распрямить спину, не
сутулиться Сделать максимально возможный вдох, не форсируя Удерживая пикфлоуметр
горизонтально, плотно обхватить мундштук губами и максимально сильно и быстро
выдохнуть Запомнить полученное значение и снова установить значение указателя на ноль
Повторить процедуру еще дважды Из трех значений выбрать максимальный, который нужно
записать в таблицу наблюдения.

12.ЭКГ Подготовка к электрокардиографии Специалист должен сохранить необходимые


персональные данные пациента с указанием даты и времени регистрации ЭКГ, а также номер
истории болезни. Подготовка предполагает правильное расположение пациента во время
процедуры и подключение электродов. Пациенту следует лечь на кушетку. В редких случаях
при определенных заболеваниях или травмах допустимо положение сидя. После того, как
пациент примет нужное положение, медсестра очистит те участки кожи, где будут
зафиксированы электроды. Для этого она использует специальное средство для
обезжиривания. Каждый электрод обрабатывается раствором хлорида натрия. На них
наносится особый гель в целях улучшения проводимости. Электроды для стандартных
отведений имеют особые цвета: Права рука – красный электрод Левая рука – желтый
электрод. Правая нога – черный электрод. Левая нога – зеленый электрод. Правильная
фиксация электродов После фиксации электродов на конечностях, проводится наложение
электродов на определенные области грудной клетки пациента: Электрод V1 – область на
уровне четвертого межреберного промежутка с правой стороны от грудины Электрод V2 –
область на уровне четвертого межреберного промежутка с левой стороны от грудины
Электрод V4 фиксируется раньше, чем V3. Электрод V4 должен быть прикреплен в области
пятого межреберного промежутка с левой стороны, при этом учитывается среднеключичная
линия Электрод V3 – область между электродами V2 и V4. Электрод V5 – область пятого
межреберного промежутка, при этом учитывается передняя подмышечная линия.Электрод
V6 – область на уровне пятого межреберья (по центру подмышечной линии). Регистрация
отведений При регистрации стандартных отведений расположение электродов должно быть
следующим: Первое стандартное отведение: правая рука (-), левая рука (+) Второе

31
стандартное отведение: правая рука (-), левая нога (+) Третье стандартное отведение: левая
нога (+), левая рука (-) Пациент должен находиться в состоянии покоя. Специалист просит
его не нервничать и не двигаться. Затем осуществляется запись ЭКГ в I, II и III отведениях,
после этого запись в усиленных и грудных отведениях. При этом в каждом отведении
регистрируется от 4 сердечных циклов.

13.водный баланс, определение отёков Отеки определяются: · путем надавливания


пальцем на кожу: если они имеются, в месте надавливания остается ямка, которая
длительное время не исчезает; · подсчетом водного баланса (ведется учет выпитой жидкости
и количества выделенной мочи) Суточный водный баланс – это соотношение между
количеством выделенной жидкости из организма и количеством введенной жидкости в
течение суток.Учитывается жидкость, содержащаяся во фруктах (условно фрукты и овощи
считаются продуктами на 100% состоящими из воды и т.о. количество жидкости,
получаемой с ними, определяется их массой), первые блюда (одна тарелка супа – 400-450 мл,
из них ½ жидкая часть, ½ густая часть), третьи блюда – 100% жидкость и считаются
парентерально введенные растворы. Для учета выпитой больным воды утром ему дается
графин с водой, из которого пьет воду только он. · при взвешивании пациента (в случае
скопления жидкости в организме резко увеличивается масса тела); В зависимости от
локализации отека различают · гидроторакс (скопление жидкости в плевральной полости); ·
гидроперикард (скопление жидкости в полости перикарда); · асцит (скопление жидкости в
брюшной полости); · анасарку (общий отек туловища). При хронических заболеваниях
сердечно-сосудистой системы отеки появляются обычно к концу дня. Если пациент ходит
(вертикальное положение туловища), отеки локализуются на нижних конечностях, при
постельном режиме – в области поясницы.

14.РГ ЖКТ Происходит ввод контрастного вещества перос, прежде чем ввести контрастное
вещество( сульфат бария с водой) проводится очистительная клизма и на голодный желудок
делается рентген в четырёх проекциях.

15.Пероральная холецистография Холецистография - (пероральная холецистография,


концентрационная холецистография) - метод исследования желчного пузыря, основанный на
способности печени улавливать из крови некоторые органические соединения йода и
выделять их с желчью, а так же на способности желчного пузыря концентрировать его
содержимое. Показания: желчекаменная болезнь, бескаменный холецистит, дискинезии
желчного пузыря. Противопоказания: тяжелые поражения печени, почек, выраженный
тиреотоксикоз, непереносимость препаратов йода. Контрастные вещества: холевид,
йопагност, билимин, осбил и др. Препараты в форме таблеток предназначены
исключительно для приема внутрь и содержат 66-77% йода. Препараты мало токсичны,
достаточно высокое содержание йода обеспечивает надежное контрастирование желчного
пузыря. Максимальная концентрация контрастных веществ в желчном пузыре обычно
наступает через 12-14 часов после их приема. Методика: однократный вечерний прием
контрастного вещества - больной накануне вечером принимает 3 г контрастного вещества,
запивая его сладким чаем, минеральной водой. После приема препарата больному
разрешается только питье чая или воды. Обычно рекомендуется дробный прием
контрастного вещества: с 19-00 по 0.5 через 15-минутные интервалы. Утром натощак, т.е.
через 12-15 часов после приема контрастного вещества, приступают к исследованию в
рентгеновском кабинете. Снимки желчного пузыря (область правого подреберья) выполняют
при вертикальном положении больного в прямой передней проекции без компрессии и с
32
компрессией. После этого переходят к изучению моторно-эвакуаторной функции желчного
пузыря. Для этого больной принимает 2 сырых яичных желтка, являющихся сильным
желчегонным раздражителем, Снимки выполняют через 30-минутные интервалы до явного
расслабления пузыря. Недостатком концентрационной холецистографии является отсутствие
изображения на снимках желчевыводящих протоков, что обусловлено недостаточным
содержанием йода в протоковой желчи. Методика двукратного вечернего приема
контрастного вещества - больной принимает две дозы контрастного вещества: 3 г с 19 до 20
часов и еще Зге 22 до 23 часов (по 0,5 каждые 10 мин). Дальнейшее исследование проводят
так же, как и при методике однократного приема. Обычно методика двукратного приема
используется при исследовании полных больных и при отсутствии на снимках тени
желчного пузыря в первый день исследования.

16.очистительная клизма Клизма - это введение жидкости в нижний отдел толстого


кишечника с лечебной или диагностической целью. В зависимости от цели различают два
вида клизм: освобождающие (очистительная, послабляющая, сифонная) и вводящие
(лекарственная, питательная). Постановка очистительной клизмы. Цель. Освобождение
кишечника от каловых масс и газов. Показания к постановке очистительной клизме: запор
(задержка стула более 48 ч); подготовка к операциям, родам; подготовка к
рентгенологическому исследованию ЖКТ, пояснично-крестцового отдела позвоночника,
костей таза, органов малого таза, органов мочевыделения; отравления; перед постановкой
лекарственных и питательных клизм. Противопоказания. Кровоточащий геморрой; острые
воспалительные заболевания прямой кишки и анального отверстия; выпадение прямой
кишки; желудочные и кишечные кровотечения; опухоли прямой кишки. Оснащение. Кружка
Эсмарха с резиновой трубкой; штатив для подвешивания кружки Эсмарха; стерильный
наконечник из пластмассы или эбонита; зажим; подкладная клеенка, судно; вазелин;
резиновые перчатки; кипяченая вода 1,5 - 2,0 л комнатной температуры; термометр для
воды; емкость с маркировкой «для использованных наконечников» с 3% раствором
хлорамина; кушетка; таз. Постановка очистительной клизмы, техника выполнения. 1. В
кружку Эсмарха наливают 1,5 - 2,0 л воды комнатной температуры 18 - 22 "с, выпускают
воздух из системы, накладывают зажим. Кружку вешают на штатив на высоте 1,0-1,5 м над
кушеткой. 2. Надевают наконечник на свободный конец резиновой трубки, смазывают его
вазелином. 3. На кушетку стелят клеенку. 4. Пациенту предлагают лечь на левый бок,
просят согнуть ноги в коленях и тазобедренных суставах. 5. Надевают перчатки, раздвигают
левой рукой ягодицы, осматривают анальное отверстие и осторожно вводят наконечник в
прямую кишку легкими вращательными движениями на глубину 3 - 4 см в направлении к
пупку, а затем до 8 - 10 см параллельно копчику. 6. Снимают зажим на резиновой трубке и
медленно вливают в просвет прямой кишки 1-2 л воды. 7. Чтобы в кишечник не попал
воздух, необходимо оставить на дне кружки небольшое количество воды. 8. Перед
извлечением наконечника на трубку накладывают зажим. 9. Вращательными движениями
осторожно извлекают наконечник в обратной последовательности (сначала параллельно
копчику, а затем от пупка). 10. Пациенту рекомендуют задержать воду на 10 - 15 мин. Для
этого ему предлагают лечь на спину и глубоко дышать. 11. Использованные наконечники
замачивают в 3% растворе хлорамина на 1 ч, затем проводят предстерилизационную
очистку, стерилизацию.

17. Сбор мочи для общего анализа, проба нечипоренко, Земницкого Проба Нечипоренко.
Перед сбором мочи надо произвести гигиенический туалет наружных половых органов,

33
затем собрать среднюю порцию утренней мочи, выделенную сразу же после сна. Для этого
первое количество выделенной мочи (15-20 миллилитров) пропускают, а среднюю порцию
мочи помещают в подготовленный чистый и сухой контейнер. Собранная моча должна быть
доставлена в лабораторию в течение 1-2 часов. Анализ мочи готовится 1 рабочий день.
Общий анализ Чтобы правильно собрать анализ мочи, накануне не рекомендуется: резко
изменять режим питья, принимать антибиотики или уросептики, в течение 12 часов до сбора
жить половой жизнью, употреблять продукты, изменяющие цвет мочи (свеклу, чернику,
морковь, ревень, спаржу и некоторые другие). Некоторые лекарства и витаминные
комплексы могут менять окраску и состав мочи, поэтому необходимо предупредить врача о
том, что Вы принимаете. Собранный в период менструации анализ также может быть не
информативным. Перед сбором необходимо привести в порядок наружные половые органы
при помощи специальных средств для интимной гигиены. Нельзя использовать
дезинфицирующие и антибактериальные вещества. Правильная гигиена поможет исключить
загрязнение мочи и попадание в анализ слизи. Для общего анализа моча собирается сразу
после пробуждения, натощак. Непосредственно перед процедурой необходимо провести
туалет наружных половых органов. Промежуток между утренним и предыдущим
мочеиспусканием должен быть около 6 часов. В лаборатории материал должен оказаться не
позднее, чем через 2 часа после сбора, так как при длительном стоянии в моче образуются
соли, и она становится непригодной для анализа. Эти правила относятся ко всем видам
исследований мочи.
Проба Земницкого Этот анализ назначают при подозрении на возможное развитие почечной
недостаточности. С помощью анализа мочи по Зимницкому можно определить способность
почек контролировать концентрацию мочи в течение суток. Правила сбора Для сбора
материала берут 8 емкостей на каждые 3 часа. Сбор мочи происходит в строго определенное
время. Даже ночью необходимо вставать по будильнику. Утром надо опорожнить мочевой
пузырь в 6 часов в унитаз. Все последующие трехчасовые порции в контейнеры. На баночках
фиксируют время 9 часов, 12, 15, 18, 21, 24, 3 часа и 6 утра. Параллельно со сбором мочи
подсчитывают количество употребляемой жидкости. Все 8 баночек утром доставляют в
лабораторию.

18. Сбор венозной крови для общего анализа Кровь для клинического анализа берут у
пациента из пальца или вены. Венозная кровь считается лучшим материалом для
исследования. Кровь следует брать натощак (после примерно 12 часов голодания,
воздержания от приема алкоголя и курения), между 7 и 9 часами утра, при минимальной
физической активности непосредственно перед взятием (в течении 20-30 мин.), в положении
пациента сидя или лежа.

19.сбор каловых масс для копрокультуры, копрокультуры, определение скрытой крови


для паразитологическоно анализа. Сбор кала осуществляется самим пациентом. Кал для
исследования должен быть собран в чистую, сухую, широкогорлую посуду, желательно
стеклянную. Перед исследованием отменяется прием медикаментов, влияющих на
секреторные процессы, а также усиливающие перистальтику кишечника. Нельзя направлять
кал на исследование после клизм, рентгенологического исследования желудка и кишечника
(примесь бария); исследование кала желательно проводить не ранее, чем через 2 суток после
рентгенологического исследования. При исследовании кала, основной целью которого,
является определение функциональной способности пищеварительного тракта, необходимо в
течение 4-5 дней соблюдать специальную унифицированную диету, содержащую

34
установленное количество различных пищевых продуктов (диета Шмидта и Певзнера).
Условия хранения и доставки материала. Кал должен быть доставлен в лабораторию и
исследован не позднее, чем через 8 – 12 часов после дефекации. Хранить до исследования
необходимо на холоду при температуре 3 – 5 С (в холодильнике).

20. Методы профилактики контрактур, тромбозов, застойной пневмонии у лежачих


больных. Профилактика контрактур. Контрактуру легче предупредить, чем вылечить. Для
предупреждения развития контрактур необходимо: как можно раньше начинать проводить
гимнастику в виде активных и пассивных упражнений, затрагивающих, по возможности, все
суставы, особенно те, которые находятся в малоподвижном состоянии. При этом следует
избегать грубых насильственных пассивных движений, вызывающих боль и рефлекторный
мышечный спазм; обеспечить правильное положение конечности (в положении,
соответствующем среднему физиологическому) при параличе мышц или в случае
иммобилизации конечности гипсовой повязкой; проводить мероприятия, направленные на
уменьшение боли в суставах и прилегающих частях конечности. Контрактуры суставов и их
профилактика существуют ортопедические приспособления - ортезы. Они поддерживают
конечность в правильном физиологическом положении. Для профилактики контрактуры
суставов кисти нужно не позволять пальцам сжиматься в кулак. Можно применить
специальный конус, обшитый мягкой тканью, лучше овечьей шерстью. Или дать больному в
руку обычный теннисный мяч. Для профилактики контрактуры суставов стопы нужно
освободить стопу от давления• Для профилактики контрактуры суставов стопы Для
профилактики контрактуры суставов стопы одеяла, и придать ей положение под углом 90
градусов. Для этого можно подоткнуть под ноги больного подушку, либо коробку. Можно
воспользоваться ортезом. Как предотвратить образование сгустков крови? Основные советы
по профилактике следующие: -Нормализовать физическую нагрузку. Каждый день делать
упражнения, чтобы привести сосуды в тонус и восстановить кровообращение. Если работа
сидячая, то стараться утром и вечером делать зарядку или физкультуру, заняться
гимнастикой или посещать спортзал. -Чаще бывать на свежем воздухе. Так кровь
обогащается кислородом, что благоприятно влияет на работу всего организма. -Полный
отказ от вредных привычек. Алкоголь и табак разрушают стенки сосудов, делают их
тонкими и хрупкими. -Не носить узкую, сдавливающую одежду. В передавленной области
могут травмироваться капилляры и скопиться тромбоциты. -Сменить привычное питание,
соблюдать специальную диету. Продукты должны быть специально подобраны, учитывая
индивидуальные особенности вашего организма и конкретно вашего случая образования
тромба. -Приём медикаментов для разжижения крови и улучшения состояния сосудов, их
укрепления. Как правило, назначают комплекс витаминов и антикоагулянты.
Лицам, которые длительно находятся на постельном режиме, следует уделять должное
внимание предупреждению застойной пневмонии. Ослабленным пациентам требуется
организация сбалансированного, разнообразного и витаминизированного питания.

21.инсулинотерапия Цель:лечебная. Показания определяет врач, чаще при заболевании


сахарным диабетом. Противопоказания определяет врач. ПОДГОТОВКА ПАЦИЕНТА: -
предупредите пациента о назначенном лечении инсулином, - предупредите пациента о месте,
времени инъекции, если пациент на общем режиме, - предупредите пациента, что сразу
после инъекции ИНСУЛИНА ОБЯЗАТЕЛЬНО надо принять пищу! ДОЗУ ИНСУЛИНА
РАССЧИТЫВАЕТ ВРАЧ! МЕСТА ВВЕДЕНИЯ ИНСУЛИНА: - передняя брюшная стенка, -
передне-наружная поверхность бедер, - подлопаточная область. ПОСЛЕДОВАТЕЛЬНОСТЬ

35
ДЕЙСТВИЙ: 1. Вымойте руки, высушите, НАДЕНЬТЕ СТЕРИЛЬНЫЕ ПЕРЧАТКИ. 2.
Приготовьте необходимый стерильный материал и инструмент (со стерильного стола, из
сухожарового шкафа, биксов и т.д.). 3. Пригласите пациента или доставьте все необходимое
для инъекции в палату. 4. Спросите у пациента разрешение на проведение инъекции. 5.
Освободите место для инъекции (пациент ОБЯЗАТЕЛЬНО ЛЕЖИТ или сидит)и ДВАЖДЫ
обработайте его ватным шариком(стерильным),смоченным спиртом. 6. Остаток спирта
снимите сухим стерильным шариком или подождите, пока кожа не станет СУХОЙ! 7.
Подкожно (под углом 30-40 градусов) введите инсулин, приложите стерильный ватный
шарик, МЕСТО ИНЪЕКЦИИ НЕ МАССИРУЙТЕ! Шприц держите в направлении СВЕРХУ
ВНИЗ, для точности введения назначенной дозы.

22.мытье и дезинфекция рук. Цель: удаление загрязнений и снижение количества


микроорганизмов до безопасного уровня (профилактика ИСМП). Показания: перед
непосредственным контактом с пациентом; после контакта с неповрежденной кожей
пациента; перед выполнением различных манипуляций по уходу за пациентом; после
контакта с биологическими средами организма, слизистыми оболочками, повязками; после
контакта с медицинским оборудованием и другими объектами, находящимися в
непосредственной близости от пациента; после лечения пациента с гнойными
воспалительными процессами; после каждого контакта с загрязненными поверхностями и
оборудованием. Противопоказания: индивидуальная непереносимость используемого мыла
или кожного антисептика. Условия эффективности: коротко постриженные ногти;
отсутствие лака на ногтях; отсутствие искусственных ногтей; отсутствие ювелирных
украшений на руках (колец, перстней и т.д); обеспечение в достаточном количестве
эффективными средствами для мытья и обеззараживания рук, а также средствами для ухода
за кожей рук (кремы, лосьоны, бальзамы). Гигиеническая обработка рук кожным
антисептиком. Оснащение: кожный антисептик, разрешенный к применению в локтевом
дозаторе (или другом бесконтактном) или в индивидуальной ёмкости. Алгоритм
манипуляции: 1. Подготовка к процедуре 1.1. Проверить условия необходимые для
эффективной обработки рук кожным антисептиком. 1.2. Приготовить всё необходимое. 2.
Выполнение процедуры 2.1. Нанесите на сложенную горсткой ладонь кожный антисептик в
количестве, рекомендованном инструкцией к применению. 2.2. Тереть ладонью о ладонь. 2.3.
Правой ладонью тереть по тыльной стороне левой кисти и наоборот. 2.4. Обработать
межпальцевые промежутки: тереть ладони со скрещенными растопыренными пальцами. 2.5.
Соединить пальцы в «замок», тереть тыльной стороной согнутых пальцев по ладони другой
руки. 2.6. Тереть поочередно круговыми движениями большие пальцы рук. 2.7. Тереть
разнонаправленными круговыми движениями поочередно ладонь кончиками пальцев
противоположной руки. 3. Окончание процедуры 3.1. Растереть раствор антисептика до суха.

23. Определение гликемии экспресс тестом 1. Последний приём пищи должен быть не
позднее 18 ч. накануне; 2. Утром до приёма пищи, воды (или любой другой жидкости) и
чистки зубов необходимо провести процедуру измерения сахара в крови, соблюдая правила
измерения. Лучше всего использовать для измерений кровь из пальца, т. к. циркуляция крови
там выше, чем в альтернативных точках измерения, как, например, плече, предплечье, бедре
или икре. Если у Вас есть проблемы с кровообращением рук, помассируйте пальцы перед
тем, как помыть их. Это же касается и измерения в альтернативных местах тела. Перед
измерением убедитесь, что код на флаконе с тест-полосками совпадает с кодом на дисплее
глюкометра. Если это не так, то перекодируйте прибор. Перед отбором крови , по

36
возможности, мойте руки теплой водой. Это служит не только гигиене, но и повышает
кровообращение. При недостаточном кровообращении взятие крови затруднено, т. к. для
получения капельки крови прокол должен быть глубже. Тщательно просушите руки. Место
прокалывания не должно быть влажным, т. к. жидкость разбавляет пробу крови, что также
ведет к неверным результатам измерения. Регулярно меняйте участки забора крови. Если
часто прокалывать одно и то же место, возникнет раздражение и утолщение кожи, и
получение крови станет более болезненным. Рекомендуется использовать 3 пальца на
каждой руке (обычно не прокалывают большой и указательный пальцы). Прокалывание
наименее болезненно, если Вы берете кровь не прямо из центра подушечки пальца, а слегка
сбоку. Не прокалывайте палец глубоко. Чем глубже прокол, тем больше повреждение
тканей, выберите оптимальную глубину прокола на ручке для прокалывания. Для взрослого
человека это уровень 2-3. Никогда не применяйте ланцет, который использовал кто-то
другой! Потому, что одна небольшая капля крови, оставшаяся на этом устройстве, в случае,
если она инфицирована, может вызвать заражение. Выдавите первую капельку крови и
уберите ее сухой ваткой. Следите за тем, чтобы кровь оставалась каплеобразной и не
смазывалась. Смазанная капля не может быть впитана тест-полоской. Не сжимайте палец,
чтобы получить большую каплю крови. При сжатии кровь смешивается с тканевой
жидкостью, что может вести к неверным результата измерений. Учтите: отверстия для
забора крови находятся по краям тест-полоски, а не на плоскости. Поэтому подведите палец
к краю тест-полоски слева или справа, они помечены чёрным цветом. Под действием
капиллярных сил, необходимое количество крови втягивается автоматически. Вынимайте
тест-полоску из упаковки непосредственно перед измерением. Тест-полоски чувствительны
к влаге. Тест-полоски можно брать сухими и чистыми пальцами в любом месте. Упаковка с
тест-полосками всегда должны быть плотно закрыта. Она имеет покрытие, поддерживающее
тест-полоски сухими. Поэтому ни в коем случае не перекладывайте тест-полоски в другую
емкость. Храните тест-полоски при нормальной комнатной температуре. Температура
хранения составляет +4 - +30 °C. Не используйте тест-полоски по истечении срока годности,
обозначенного на упаковке.

24. Уход за ротовой и носовой полостями у больного без сознания Уход за носом
Проводите процедуру ежедневно, а также по мере необходимости. Вам понадобится: Ватные
палочки или турунды из марлевых салфеток. Стерильные турунды можно приобрести в
аптеке или сделать их из чистых ватных дисков: возьмите половинку ватного диска и
накрутите его на конец зубочистки, затем уберите зубочистку и дополнительно уплотните
турунду скручиванием. Вазелиновое масло (или оливковое/ детское увлажняющее масло),
Полотенце или пеленка. Вымойте руки и наденьте перчатки. Наденьте фартук на больного.
Смочите ватную палочку (турунду) во флаконе с вазелиновым маслом. Палочка (турунда)
должна быть смочена равномерно и умеренно, чтобы масло не капало с нее. Возьмите
палочку или турунду в правую руку. Левой приподнимите кончик носа пациента и
аккуратно, вращательными движениями введите палочку (турунду) в ноздрю. Если в носу у
больного человека сухие корочки, подержите палочку (турунду) 1-2 минуты для их
размягчения. Достаньте ватную палочку вращательными движениями. Возьмите чистую
палочку (турунду), смочите во флаконе с вазелиновым маслом и повторите процедуру до
полного удаления корочек. Обработайте другой носовой ход аналогичным способом.
Уход за полостью рта. Проводите процедуру ежедневно утром и вечером, а также после
каждого приема пищи и после приступов рвоты. У тяжелобольных людей возрастает риск
возникновения инфекций полости рта, поэтому требуется постоянный уход. Вам
37
понадобится: Щетка и шпатель для полости рта (можно купить в аптеке). Марлевые
салфетки. Свежевыжатый лимонный сок с глицерином (1:2). Емкость. Непромокаемые
пеленка или полотенце. Гигиеническая помада. Заменить шпатель, марлевые салфетки и
лимонный сок с глицерином можно специальными палочками по уходу за полостью рта –
продаются в аптеке. Поверните больного на бок так, чтобы его лицо находилось на краю
подушки. Под его головой и на грудной клетке расстелите впитывающую пеленку. Поставьте
емкость под подбородок. Смочите зубную щетку лимонным соком с глицерином. Попросите
приоткрыть рот или бережно, не применяя силы, сделайте это сами с согласия больного.
«Выметающим» движением сверху вниз проведите по нижней и верхней челюстям человека.
Повторите несколько раз в течение 30-40 секунд. Оберните шпатель марлевой салфеткой и
смочите его в лимонном соке с глицерином. Аккуратно обработайте внутреннюю
поверхность щек, язык, подъязычную область, твердое небо. Если на языке есть налет, его
необходимо очистить мягкой щеткой или шпателем с марлевой салфеткой. Намажьте губы
человека гигиенической помадой (по желанию).

25. смена постельного белья Цель. Создание постельного комфорта (одно из мероприятий
лечебно-охранительного режима); профилактика пролежней; соблюдение личной гигиены
пациента. Показания. Постельный режим пациента. Оснащение. Чистая простыня,
достаточно большая по размеру, без швов, заплат; чистый пододеяльник; две наволочки.
Техника выполнения: Продольный способ смены постельного белья тяжелобольному: -
применяется, когда пациента можно повернуть на бок. 1. Чистую простыню скатывают
валиком по длине на 2/3. 2. Убирают одеяло, осторожно приподнимают голову пациента и
убирают подушки. 3. Поворачивают пациента на бок от себя. 4. На освободившейся
половине постели скатывают грязную простыню валиком к середине кровати (под пациента).
5. На освободившуюся часть кровати раскатывают подготовленную чистую простыню
валиком к пациенту. 6. Поворачивают пациента на другой бок лицом к себе. 7. Убирают
грязную простыню с освободившейся части кровати, расправляют чистую, натягивают и
заправляют со всех сторон под матрас. 8. Укладывают пациента на спину, подкладывают
подушки в чистых наволочках. 9. Сменяют пододеяльник, укрывают пациента одеялом.
Поперечный способ смены постельного белья тяжелобольному: - применяется, когда
пациента нельзя повернуть на бок, но можно посадить или приподнять верхнюю часть
туловища. 1. Простыню скатывают валиком по ширине на 2/3. 2. Просят санитарку
приподнять пациента, поддерживая его за спину и плечи. 3. Убирают подушки, скатывают
валиком грязную простыню к спине пациента. 4. Чистую простыню раскатывают валиком к
спине пациента. 5. Подкладывают подушки в чистых наволочках, опускают пациента на
подушки. 6. Просят санитарку приподнять пациента в области таза. 7. Скатывают грязную
простыню с освободившейся части кровати и раскатывают чистую, кладут пациента. 8.
Просят санитарку приподнять ноги пациента. 9. Убирают с кровати грязную простыню,
раскатывают до конца чистую. 10. Чистую простыню со всех сторон подворачивают под
матрац. 11. Меняют пододеяльник, укрывают пациента.

26. Уход при пролежня . Размещение пациента на функциональной кровати (в условиях


больницы). Должны быть поручни с обеих сторон и устройство для приподнимания
изголовья кровати. Пациента нельзя размещать на кровати с панцирной сеткой или со
старыми пружинными матрацами. Высота кровати должна быть на уровне середины бедер
ухаживающего за пациентом. 2. Пациент, перемещаемый или перемещающийся в кресло,
должен находиться на кровати с изменяющейся высотой, позволяющей ему самостоятельно,

38
с помощью других подручных средств перемещаться из кровати. 3. Выбор
противопролежневого матраца зависит от степени риска развития пролежней и массы тела
пациента. При низкой степени риска может быть достаточно поролонового матраца
толщиной 10 см. При более высокой степени риска, а также при имеющихся пролежнях
разных стадий нужны другие матрацы. При размещении пациента в кресле (кресле-каталке)
под ягодицы и за спину помещаются поролоновые подушки, толщиной 10 см. Под стопы
помещаются поролоновые прокладки, толщиной не менее 3 см. 4. Постельное белье –
хлопчатобумажное. Одеяло – легкое. 5. Под уязвимые участки необходимо подкладывать
валики и подушки из поролона. 6. Изменение положения тела осуществлять каждые 2 часа, в
т. ч. в ночное время, по графику: низкое положение Фаулера, положение «на боку»,
положение Симса, положение «на животе» (по согласованию с врачом). Положение Фаулера
должно совпадать со временем приема пищи. При каждом перемещении - осматривать
участки риска. Результаты осмотра - записывать в лист регистрации противопролежневых
мероприятий. 7. Перемещение пациента осуществлять бережно, исключая трение и сдвиг
тканей, приподнимая его над постелью, или используя подкладную простыню. 8. Не
допускать, чтобы в положении «на боку» пациент лежал непосредственно на большом
вертеле бедра. 9. Не подвергать участки риска трению. Массаж всего тела, в т. ч. около
участков риска (в радиусе не менее 5 см от костного выступа) проводить после обильного
нанесения питательного (увлажняющего) крема на кожу. 10. Мытье кожи проводить без
трения и кускового мыла, использовать жидкое мыло. Тщательно высушивать кожу после
мытья промокающими движениями. 11. Использовать непромокаемые пеленки и
подгузники, уменьшающие чрезмерную влажность. 12. Максимально расширять активность
пациента: обучить его самопомощи для уменьшения давления на точки опоры. Поощрять его
изменять положение: поворачиваться, используя поручни кровати, подтягиваться. 13.
Научить родственников и других лиц, осуществляющих уход, уменьшать риск повреждения
тканей под действием давления: -регулярно изменять положение тела; -использовать
приспособления, уменьшающие давление (подушки, поролон, прокладки); -соблюдать
правила приподнимания и перемещения: исключать трение и сдвиг тканей; -осматривать всю
кожу не реже 1 раза в день, а участки риска - при каждом перемещении; -осуществлять
правильное питание и адекватный прием жидкости; -правильно осуществлять гигиенические
процедуры: исключать трение. 14. Не допускать чрезмерного увлажнения или сухости кожи:
при чрезмерном увлажнении - подсушивать, используя присыпки без талька, при сухости -
увлажнять кремом. 15. Постоянно поддерживать комфортное состояние постели: стряхивать
крошки, расправлять складки. 16. Обучить пациента дыхательным упражнениям и поощрять
его выполнять их каждые 2 часа.

39

Вам также может понравиться